Last visit was: 24 Apr 2024, 13:09 It is currently 24 Apr 2024, 13:09

Close
GMAT Club Daily Prep
Thank you for using the timer - this advanced tool can estimate your performance and suggest more practice questions. We have subscribed you to Daily Prep Questions via email.

Customized
for You

we will pick new questions that match your level based on your Timer History

Track
Your Progress

every week, we’ll send you an estimated GMAT score based on your performance

Practice
Pays

we will pick new questions that match your level based on your Timer History
Not interested in getting valuable practice questions and articles delivered to your email? No problem, unsubscribe here.
Close
Request Expert Reply
Confirm Cancel
Magoosh GMAT Instructor
Joined: 28 Dec 2011
Posts: 4450
Own Kudos [?]: 28569 [0]
Given Kudos: 130
Magoosh GMAT Instructor
Joined: 28 Dec 2011
Posts: 4450
Own Kudos [?]: 28569 [1]
Given Kudos: 130
Magoosh GMAT Instructor
Joined: 28 Dec 2011
Posts: 4450
Own Kudos [?]: 28569 [0]
Given Kudos: 130
Magoosh GMAT Instructor
Joined: 28 Dec 2011
Posts: 4450
Own Kudos [?]: 28569 [0]
Given Kudos: 130
Computer Adaptive Testing on the GMAT [#permalink]
Expert Reply
FROM Magoosh Blog: Computer Adaptive Testing on the GMAT
What is an adaptive test? How should you change your study strategy to prepare for the GMAT Computer Adaptive Test (CAT)? Computer adaptive testing definitely puts pressure on your testing experience and forces you to adapt your test-taking strategy in small ways. It’s especially important to understand how the GMAT calculates your score and what this means for you as a test-taker.



Fact: The GMAT uses Computer Adaptive Testing (CAT)
This means, first of all, that each question you answer correctly or incorrectly determines what questions you will see later in the GMAT.  It also means that any two people, even two people of nearly identical abilities and preparedness, will not see identical questions when they take their respective GMATs. BUT, because of the magic of psychometrics, two people who perform with comparable skill & strategy & focus will have comparable GMAT scores.  The magic of how computer adaptive testing is different for everyone but the score is fair for everyone—if you don’t have a Ph.D. in Psychometrics or Statistics or something like that, then just take that magic as an article of faith.

It’s also very important to point out: only the GMAT Quantitative and Verbal sections are computer adaptive.  The Integrated Reasoning is NOT computer adaptive, and of course, the AWA essay is not.  Everything I say in this blog about the CAT concerns only the Q & V sections.

What is an adaptive test and how does the GMAT CAT work?
The GMAT is trying to figure out objectively your Quantitative Ability and your Verbal Abilities in a relatively short time.  Think of computer adaptive testing as a big “twenty questions” game.  Suppose your “opponent” picks a US city, and you are allowed to ask “horizontal yes/no questions” (e.g. “Is your city east/west of X?”) and “vertical yes/no questions” (e.g. “Is your city north/south of X?).

You might ask a bunch of horizontal questions.  It is west of Albuquerque? No.  Is it east of Atlanta? No.  Is it east of Denver? No.  Is it east of Santa Fe? No. OK, that narrows things down to a relatively thin vertical band.

Then a bunch of vertical questions.  It is North of Wichita? No.  Is it north of Birmingham, AL? No.  It is north of New Orleans? Yes.  Is it north of Tucson? No.

Among major cities, those answers are enough to home in on El Paso, TX.  Much in the same way, the GMAT asks you two question types, Math and Verbal.  By giving you easy and hard questions of each kind, the computer adaptive testing homes in on what is most appropriately your level.

How accurate is the Computer Adaptive Test?
The “twenty questions” analogy is helpful for understanding computer adaptive testing, but the latter is for more complex: after all, what the CAT is measuring is not cut-and-dry exact.  If we want to know where a city like El Paso is, that’s totally objective, and the questions about whether such-and-such city is N/S or E/W of El Paso are also totally objective.  That means, with very few yes/no questions, one could home in on an exact location.

Someone’s math & verbal ability is not so precise a thing.  First of all, there are easy questions you definitely can answer, there are super-hard questions you definitely can’t answer, but for the questions in-between, it’s gray: there’s a difficulty level at which you usually get questions right, and another slightly higher level at which you usually get the questions wrong.  For the sake of argument, let’s say that we have figured out questions that are exactly at your ability level if, on average, you get questions at that level right 50% of the time.  Clearly, whether you answer any one question correctly or not is not enough information to tell whether it’s at your ability level or not; determining your level is going to be about an average over several questions, not simply the answer to one.  Furthermore, there are frequent aberrations.

Super-brilliant people sometimes get an easy question wrong, and folks who are minimally prepared can still guess correctly on one of the toughest questions.  With statistics, the computer can absorb such aberrations.  What the computer is doing throughout your test is averaging over the difficulty ratings of all the previous questions, using the data about which you got right and which you got wrong to create a complex average that is the best estimate of your ability, and each new question it feeds you is the computer’s attempt to refine that best estimate.  This process mostly involves questions around your level, but in its sweeps, the algorithm sometimes will give you a really hard or really easy question as a check.

Your score is a composite result that takes into account the difficulty of each question you got right and the difficulty of each question you got wrong.  The exact details of the algorithm that the computer uses to do this are (a) probably incomprehensible if you don’t have a Ph.D. in Psychometrics, and (b) the secret proprietary information of GMAC.  Legally, we can’t get access to that algorithm, and even if we could, we probably wouldn’t understand it anyway. Rest assured, though: it is accurate.  🙂

What does it imply about the accuracy of the computer adaptive testing if a person takes and retakes the GMAT multiple times, getting a different score each time?  This empirically observable fact doesn’t compromise the accuracy, the statistical robustness, of the computer adaptive testing itself.  Think about it this way.

Suppose, I walk into the GMAT on the first Tuesday in March.  On that particular day, I have slept a certain amount over the nights of the preceding week.  On that particular day, I have eaten whatever foods I have eaten over the past several days, and have had whatever alcohol or other toxins during the past week.  On that particular day, I have whatever combination of life-stresses churning around in the back of my mind.  On that particular day, I may feel healthy, but my immune system is in the process of fighting off whatever it is fighting off.  On that particular day, my hormones may be at any particular level: this fluctuates somewhat more predictably in females and considerably less predictably in males.  On that particular day, I will have whatever additional stress about the test itself, about my relative level of preparedness or lack thereof, my general level of self-belief and self-confidence, etc.  All of these factors play into one’s GMAT readiness.  The GMAT accurately records how ready I was for the GMAT that particular day.

Suppose I don’t do well on that sitting, then take a course, such as the Magoosh course.  Then I schedule my second attempt, say, on the last Friday in May.  The extra experience, the deeper understanding, and the familiarity with the test-taking procedure may boost my confidence considerably.  Again, my performance will depend in part on all these other factors, but if I learned a great deal more, in all likelihood, I will be more ready for the GMAT, and the computer adaptive testing will accurately measure the GMAT readiness on this date.

Hold on!  It sounds as if Mike is saying that all the GMAT measures is how well you can take the GMAT!  Well, yes and no.  It is not a pure intelligence test, insofar as one can take a course and improve: for example, Magoosh has a 50-point score guarantee.  Also, eating well, getting adequate sleep, and practicing stress reduction all can help you put your best self forward on test day.  Nevertheless, the folks at GMAC, the place that produces the GMAT, will tell us that, as a statistical tool, the GMAT CAT is not only robust but valid.  To be statistically valid means that the test has predictive power: in particular, the GMAT is very good at predicting how one will handle the academic demands of business school.  If by studying you make yourself more prepared for the GMAT, unwittingly you have also made yourself more prepared for your future business school classes.

Facts vs. Myths about Computer Adaptive Testing on the GMAT
Fact: If you get medium questions mostly right, the computer will start to feed you harder questions; if you get medium questions mostly wrong, the computer will start to feed you easier questions.

In general, this is true: this is the overall general pattern.  The computer adaptive testing adjusts to your level throughout—much like the E/W and N/S question in the geography game above, it is constantly refining its picture of your ability, question by question.

 

Myth: If I suddenly get a ridiculously easy question, that means I got the last question wrong.

First of all, a question that seems easy to you may or may not actually be a truly “easy” question, that is, one that most people get right; it may just be that you saw a shortcut that most people don’t see.

Even if the question is, by objective standards, an easy question, no conclusion can be drawn about the previous question.  The computer adaptive testing is running a complex algorithm, which sometimes involves giving you a very easy or a very hard question.  Don’t take it personally: the adaptive testing is just running its algorithm.

 

Fact: You can get several questions wrong and still get a good score.

The computer adaptive testing has to give you several questions well above your ability, questions that you almost invariably will get wrong, in order for it to zero in on your actual ability.  You are not penalized for that: that’s just what the computer adaptive testing must do as part of its algorithm.

In fact, here’s the irony: the person who gets a 460 and the person who gets a 780 may get approximately the same number of questions right & wrong.  The number of questions correct, by itself, indicates nothing: all that matters is the difficulty of the questions.  The person who got a 460 probably got several relatively easy questions wrong but even easier question right.  The person who got a 780 got many super-hard questions correct but even harder ones wrong.   The computer adaptive testing has an extremely sophisticated way of finding your level and customizing the analysis to someone at your level.

 

Myth: The first question or first five questions are super-important, because these determine the course of easy/hard questions from there.

Totally false.  The computer adaptive testing is performing a complex process of estimation that can handle aberrations, even if one of the aberrations happens on question #1.  Don’t worry: over the course of the whole test, the computer will give you the combination of questions it must in order to determine your abilities.  Furthermore, the algorithm is such that order of the questions doesn’t affect your score at all.  If you get a certain question right then whether it was the first question, a middle question, or the last question, doesn’t matter at all.  What does matter for your score is the difficulty of the question, and whether you got it right or wrong, but not where it fell in the test.

This is a very common misconception, that for some value N, the first N questions are really important.  First of all, objectively, this is patently false, but even disregarding that, think about it.  What concrete strategy would this imply?  Would it make sense to “try harder” on the first N questions?  Wouldn’t that imply trying less hard on the rest, the majority of the test?  By approaching a section with the thought that the first N questions are somehow “the most important,” typically all the student thereby accomplishes is to generate a great deal of stress right at the beginning of the section, a result that will almost invariably have counterproductive results.

You see, as far as effort is concerned, the only strategy that makes any sense at all is to bring your absolute best effort to each and every question that you see.  That is the only strategy that makes sense, and as it happens, the computer adaptive testing is 100% designed to support such an approach.  Excellence can be consistent when it is a habit: for more on this, see the discussion of the habits of excellence.

 

Fact: Not finishing all the questions in a section hurts your score.

That is quite true.  It’s exceedingly important not only to learn content and strategy, but also to practice working efficiently so that you don’t run out of time.  Ideally, you want to hone your time management skills so that you have abundant time on even the last questions on a section.

If you are running out of time, you should understand the strategies concerning solution behavior and guessing.

 

Myth: You can outsmart the CAT. 

The algorithm is far too complex.  There’s no sense stressing about “how did I do on those questions?” or “why is it asking this kind of question now?”  Again just do your best on the question in front of you at any moment, submit it, and then forget about that question entirely.

 

Fact: Systematically reviewing math and verbal content, as well as strategies specific to each question type, can vastly enhance your GMAT score.

That is most certainly true, and that’s why Magoosh can give you such an advantage.  With a couple hundred lesson videos discussing both content and strategy, and over 800 practice questions, each with its own video explanation, you will get top-notch preparation for the GMAT at only a fraction of what you would pay for a comparable course.

 

Editor’s Note: This post was originally published in February, 2012 and has been updated for freshness, accuracy, and comprehensiveness.

The post Computer Adaptive Testing on the GMAT appeared first on Magoosh GMAT Blog.
This Blog post was imported into the forum automatically. We hope you found it helpful. Please use the Kudos button if you did, or please PM/DM me if you found it disruptive and I will take care of it. -BB
Magoosh GMAT Instructor
Joined: 28 Dec 2011
Posts: 4450
Own Kudos [?]: 28569 [1]
Given Kudos: 130
The JD/MBA Student: A Professorial Perspective [#permalink]
1
Bookmarks
Expert Reply
FROM Magoosh Blog: The JD/MBA Student: A Professorial Perspective
There are some real benefits to a dual JD/MBA, in terms of JDA MBA salary and JD MBA jobs. But beyond the MBA JD salary and jobs, is this degree worth it? As a JD/MBA professor, I’m happy to give you my perspective on this unique type of program: its structure, its costs and rewards, and the learning experience it provides.


superawesomevectors, publicdomainpictures.net

The dual-degree JD/MBA: Degree structure and top programs
Law and business graduate faculty would probably agree that this joint program is probably the most challenging in the educational community. And, from an employer’s perspective, this duality is probably the most commercially relevant.

This blog post speaks to the prospective MBA candidate who is curious about law school, the time/energy commitment, and the benefits of this joint degree, the most marketable dual degree now being offered at many graduate schools across the country.  Want a four-year adventure? Then embark upon the most challenging and rigorous graduate program available.

Typically, the first and fourth year of the program consist solely of law courses while the second and third year are an integration of law and business. (However, the Kellogg School and Northwestern University’s School of Law tout a three-year, fully integrated, joint program.)

Per U.S. News & World Report, strong rankings included the well-recognized, familiar-name schools.

Harvard: business school (1), law school (2)

Stanford: business school (2), law school (2)

University of Chicago: business school (2), law school (4)

University of Pennsylvania: business school (4), law school (7)

Northwestern University: business school (5), law school (12)

UC Berkeley: business school (7), law school (8)

Yale: business school (8), law school (1)

Columbia: business school (10), law school  (4)

JD/MBA dual degree in terms of extra cost… or savings
Tuition structures for JD/MBA dual degrees vary a lot at different universities. Generally, a JD/MBA dual degree costs more than getting just an MBA. Surprisingly, sometimes this dual degree can cost slightly less than getting a JD alone. And getting a dual JD/MBA will always cost less than completing a JD and an MBA separately.

Schemes for tuition and fees can be a bit complex, almost convoluted. To give you an example, I’ll walk you through the current tuition structure and total schooling costs of the top program in the list above: Harvard.

Harvard’s JD/MBA program takes four years to complete. In the first two years, students study for one year at the Harvard School of Business and for one year at the Harvard School of Law. During those years, JD/MBA students pay full tuition at whichever school they’re taking classes in for the year. At Harvard’s current tuition schedule, JD/MBA scholars pay $48,600 for their first year at business school and $59,550 for their first year in law school. This means a total tuition cost of $108,150 for the first half of the program.

In years 3 and 4 of the JD/MBA degree path, students study simultaneously in both schools, but take the equivalent of 1 additional year of courses in the business school and one additional year of law school classes. During these upper-level years in the degree, students pay 40% of current business school tuition rate and 56% of current law school rate. Right now, that means an additional $19,440 in tuition for business school and an extra $33,348 in law school tuition.

So the total tuition cost for Harvard’s JD/MBA is $160,938. A Harvard JD on its own takes three years, costing $178,650. The MBA at Harvard is a two year program, priced right now at $97,200 in total tuition. If you decide on a JD/MBA dual degree at Harvard but you had originally planned for a JD, you save $17,712 in tuition and spend an extra year in school. If your original choice would have been just an MBA, you spend an extra $62,738 in tuition and spend an extra two years in school. Either way, you ultimately spend more money. The $17,712 you save if you upgrade from a  JD to a JD/MBA will be eaten up by the cost of an extra year of living expenses and lost wages.

Not all universities offer Harvard’s exact arrangements. Still, JD/MBA dual degrees ultimately cost more than just a JD or just an MBA, no matter what campus you go to. At Stanford University, a JD/MBA takes four years and costs $245,238 in tuition, while a JD is 3 years long at $168,237. Then, a Stanford MBA is two years long at $133,080. And the University of Chicago estimates their total extra JD/MBA cost– in extra tuition and in lost time– to be $230,000 more than just getting a law degree and around $500,000 more than just getting an MBA.

JD MBA salary and JD MBA jobs
Is a JD/MBA dual degree worth the extra time and money, in terms of employment and pay after graduation? Again, this is a little hard to figure out. One thing that makes this a complicated question is the fact that no job really requires both a JD and an MBA. So if you get this dual degree, you’ll either end up working as a lawyer or a business manager after graduation. Obviously you won’t be holding down both of these jobs at the same time.

To get an idea of the kind of money you might make right after finishing a JD/MBA, let’s look once again at the top three schools listed at the beginning of this post. According to Forbes Magazine, Harvard MBA graduates make around $239,000 a year immediately after graduation, Stanford MBA graduates make $255K coming out the door, and University of Chicago MBA holders have average entry level pay of about $250K. Forbes also reports that those with a JD from Harvard make $130,00 a year to start, while Stanford lawyers pull in $147,000 right after school, and University of Chicago JD graduates make around $132K.

Post MBA JD salary and fringe benefits
These post-degree salary figures don’t tell the whole story, though. In the long run, JD/MBA holders have much better prospects for promotions, increases in pay, and general upward mobility. Those with both a JD and an MBA can switch between business management work and law in the course of their careers, and can use their degree to negotiate better entry level pay, better raises, and more frequent promotions.

Ultimately, dual-degree graduates may also enjoy a wider spectrum of job opportunities, often finding work in the general counsel’s office of a large corporation or in the business litigation division of a large law firm.

The JD/MBA dual degree: My perspective as a professor
From a professorial perspective the JD/MBA students, usually 12-14 in a class, were always a delight to teach. I always expected a certain level of preparedness and they consistently delivered. And they had expectations for me as well: they wanted topical, seminal law cases assigned for discussion. As a group I noticed a given student would lean toward or favor a particular discipline. Those who were more business oriented were older, more practical with considerable work experience. Those who considered themselves law students first were more irreverent, talkative, and generally better writers most often with a liberal arts undergraduate degree. And they formed tribal groups early on sometime arguing loudly across the classroom.

If you are considering the JD/MBA engage a graduate school counselor, check your finances, ponder the job market, and think carefully about a radical change of life style.

Interested in reading about a Magoosh student who got a JD before an MBA? Look no farther: Magoosh Student Post: From a JD to an MBA.

Editor’s Note: This post was originally published in March 2012 and has been updated for freshness, accuracy, and comprehensiveness.

The post The JD/MBA Student: A Professorial Perspective appeared first on Magoosh GMAT Blog.
This Blog post was imported into the forum automatically. We hope you found it helpful. Please use the Kudos button if you did, or please PM/DM me if you found it disruptive and I will take care of it. -BB
Magoosh GMAT Instructor
Joined: 28 Dec 2011
Posts: 4450
Own Kudos [?]: 28569 [0]
Given Kudos: 130
Why GMAT Practice Test Scores Vary [#permalink]
Expert Reply
FROM Magoosh Blog: Why GMAT Practice Test Scores Vary
Magoosh’s students take the best GMAT practice tests they can find. Which is the best GMAT practice test? It depends. The Manhattan GMAT practice test and ones from Magoosh and GMAC come to mind. Others too.



Once our students have taken several practice tests, it’s not uncommon for Magoosh to get student questions like this one:

I recently took a practice test and it was much lower than my other practice tests. I’ve been studying for awhile now, and I am freaking out a little bit. Shouldn’t I be improving? Here are the scores from my practice tests.

Practice test 1: 580

Practice test 2: 620

Practice test 3: 590

Practice test 4: 550

What happened? Can you help me improve? I don’t want this to happen when I take the test.

This is a common question that we encounter at Magoosh—a student is taking practice tests and their scores are all over the place. Shouldn’t there be some trend, like going up, after lots of practice? In theory, there should. But this is not always the case. Why?

This statistical variation is common and happens for a number of reasons. Let’s explore some of the reasons for this variation.

Varied Testing Conditions for GMAT Practice Tests
The overall circumstances surrounding your practice GMAT exams can cause a lot of variation in test scores. Below, we’ll look at three of the biggest circumstantial contributors to inconsistent scoring during GMAT practice:

  • Different Testing Environment
    Our surroundings have a powerful influence over us. If you take the same test twice, but change the testing environment, you can expect to see a difference in scores. The difference between sitting in your room and going to a private study room at the local library to take a practice test is dramatic. One spot is obviously more ideal than the other. And by “ideal,” I mean that it is more like the testing environment for the real exam. Did you vary your testing environment? Was your low score from the test you took while in a cafe? Look no further. You found the culprit then. Make sure that you emulate the testing environment that you will see on test day.
  • Different Diet

    What you put– or don’t put– into your body matters. Think back to what you ate in the 24 hours leading up to your GMAT practice test. Was your belly full of sweets or junk food? Did you skip a meal? Did you indulge in delectable southern BBQ? That might have been the difference. To optimize your score and your ability to succeed, you need to eat well. And not just before the test, but all the time. You need to give your brain the food it needs. Eat fish, almonds, avocados, garlic, and blueberries. Here is a great “brain food” infographic on what to eat and why.
  • Different Sleep Patterns

    Erratic and irregular sleep patterns are detrimental to learning and performance. Research into the causes of poor grades shows that a consistent eight hours of sleep is an important part of learning. Not only will it boost your focus and performance, but it will also keep you positive and foster emotional health. Make it a point to sleep enough before your test. And if you want all those daytime hours of studying to really make a difference in your GMAT scores, you need to get plenty of sleep in the weeks leading up to the test.
Different GMAT Practice Tests
The most obvious reason for inconsistent practice GMAT scores is the use of practice tests from different companies. Each test prep company creates questions of varying quality. And this quality affects students’ scores. Poorly created questions, like you might see with the Princeton Review, will not give you an accurate prediction of your score.

But getting consistent GMAT practice scores isn’t just a matter of choosing well-deigned practice tests. We also have to think about how test prep companies calculate your estimated score at the end of the test. No company knows exactly what the GMAC– the official maker of the GMAT– does to convert a raw score into a scaled score. As such, each company has to approximate this conversion, and each company does something slightly different. Since this is not standardized across practice tests, you can definitely expect to see variations in the scaled score from test to test.

How Magoosh Calculates GMAT Practice Test Scores
Like other providers of unofficial GMAT test prep, we have to do our best to approximate our students’ scores. To estimate a student’s score from a mock test, we look at how students performed on Magoosh questions and compare this to the score they get on the actual test after they practice with us. With this information, we calculate a predicted scaled score based on a student’s percent correct on Magoosh questions.

As more and more students have continued to use Magoosh and take the GMAT, our predictions have become more and more accurate. Still, we’re cautious about predicting an exact score, because the correlation between a Magoosh student’s practice scores and their real score can vary a bit. To account for this variability, we provide students with a score range rather than a single score. Sharat’s a post on how to calculate GMAT scores gives additional details about how we predict a student’s performance.

The Value of the Manhattan GMAT Practice Test
Better-designed questions with fewer mistakes, such as the ones on Manhattan GMAT practice tests, make for a more accurate assessment… but not a perfect one. (I hear grumblings in forums about inflated verbal scores on Manhattan tests). Overall, though, Manhattan GMAT practice tests and materials are highly recommended by Magoosh. Manhattan offers great support in both predicting and maximizing your GMAT score.

Not only that, but Manhattan GMAT also offers supplemental material to the ultimate, “must have” source of GMAT practice testing. This brings us to….

The Best GMAT Practice Test
The best GMAT practice test you can get for free is the one you can download from GMAC itself. If you want the best prediction of your score on the test, use this exam and other practice testing materials from the people who make the real GMAT. Only official GMAT materials are guaranteed to completely capture the true nature of the exam. You can go to MBA.com’s GMAT store catalog for the newest official GMAT materials. Of course, you don’t necessarily need to buy the most recent GMAT books. Be sure to check websites like Amazon.com for additional, slightly older official GMAT books. Anything from 2013 or later will be accurate to the format of the current test.

Score Variation & The Best GMAT Practice Test: Takeaway
Variation in practice test scores (even from the best, most official materials) is normal and expected. Many factors affect the predicted score you receive on a practice test. Ultimately, don’t invest too much in the estimated scores. Remember that it is only a predication and not necessarily accurate. Use practice tests as a way to simulate the test, not as a untainted measure of your GMAT score.

Editor’s Note: This post was originally published in December 2013 and has been updated for freshness, accuracy, and comprehensiveness.

The post Why GMAT Practice Test Scores Vary appeared first on Magoosh GMAT Blog.
This Blog post was imported into the forum automatically. We hope you found it helpful. Please use the Kudos button if you did, or please PM/DM me if you found it disruptive and I will take care of it. -BB
Magoosh GMAT Instructor
Joined: 28 Dec 2011
Posts: 4450
Own Kudos [?]: 28569 [0]
Given Kudos: 130
Exam Lifehacks Infographic [#permalink]
Expert Reply
FROM Magoosh Blog: Exam Lifehacks Infographic
Magoosh is happy to present a life hacks blog post to help you maximize your GMAT performance. These GMAT hacks work– and they can be surprisingly easy!

Sometimes the simplest, smallest study tricks make the biggest difference. They can boost your confidence, improve your focus, and dissipate your exam anxiety. They can even open your eyes to a new method of organization that will boost your academics for years to come.

Today, we share our favorite unexpected lifehacks to improve your score. Enjoy!

Note: For sharing purposes, all of the images in this GMAT life hacks blog post can also be found as a single image at the end of this post, with an embed code for sharing the image. Spread the word!



GMAT Hacks 1-6: When you have one month left before the test
  • Music
    Music can amplify the good results of your GMAT study sessions. Listening to music can calm your test prep nerves. The right kind of music will help you focus on your GMAT activities with clarity. But you really do need to pick the right kind of music. Research shows that if you listen to music while you study, you should avoid certain kinds of music. Music with lyrics can be distracting, and so can music that has abrupt changes in tempo and is not “calm.” See Magoosh’s recommended genres of music in the graphic below.
  • The Pomdoro Technique
    The Pomdoro Technique is a timing scheme for long study sessions. The Pomdoro timetable  can get you into a good rhythm of study. With Pomodoro timing, you prepare for the GMAT for 25 minutes without interruption, and then you take a five minute break to relax. The creator of this technique calls the 25 minute work/5 minute break cycle a “pomdoro.” In this technique, you also take an extra 15 minute break after completing 4 pomdoros. This approach to time management breaks up your study into  manageable segments, while still keeping you “on task” most of the time. (Native Italian speakers may recognize that “pomdoro” means tomato. This is because the technique was originally developed by Italian learning specialist Francesco Cirillo. For more information, see Cirillo’s official Pomdoro Technique website.)
  • Distraction Apps
    The urge to pick up your smart phone, go online, play electronic games, etc. is perhaps the biggest killer of productivity in the modern world. It can be hard to focus on your GMAT prep when so many other interesting electronic distractions are at your fingertips. Fortunately, there are ways to fight mobile distraction, using mobile technology. With the SelfControl App, you can block specific distracting websites for up to 24 hours at a time. And with ColdTurkey, you can actually schedule website blockage, so that your favorite and most distracting Internet destinations are unavailable during scheduled study times. Then there’s the Freedom App. This app blocks the Internet and other phone apps altogether. (If you want to do this and also block calls, try a combination of the Freedom App and your phone’s airplane mode.)
  • The Seinfeld X Technique
    Comedy is a high pressure and surprisingly serious business. Comedians need to come up with new jokes every day. To motivate himself to always come up with new ideas, famed comedian Jerry Seinfeld would put a big red X over each day on his wall calendar– provided he spent time writing jokes on that day. Visually seeing a chain of red Xs can be very motivating. You’ll be motivated to not break the chain. (Except of course, on a designated day or two each week where you take a break from study.) For your version of the Seinfeld X Technique, put a red X on any day where you meet all of your GMAT prep goals.
  • Light exercise

    I don’t know about you, but when I sit down for too long, I start to get antsy. My mind wanders from the task at hand, and I want to move around. Following the urge to move can be good. Vent that excess energy and you’ll be able to sit back down again and focus on your studies. What’s important here is that you keep the exercise light. A 20 minute jog or a few dozen push-ups will wear you out. But a good, brief break for physical activity will vent your excess energy and allow you to return to your GMAT studies with greater focus. You can see our list of recommended light exercises in the graphic below.
  • Change of scenery
    Psychological research shows that you learn more from your studies if you study in a  few different places.  In fact, one study by psychologist Robert Bjork show that college students who review vocabulary in two different rooms recall 20% more words than students who have just one study spot. Of course, you need to choose your varied study spaces intelligently. Don’t choose something too comfortable. A soft chair or big comfy couch can soothe you right to sleep. And obviously, you should avoid study spaces that are noisy and distracting.


GMAT Hacks 7-12: Thinks you can do the day before the test
Once you get to the day before the test, the life hacks for good GMAT performance get even simpler. Now you no longer need to work about study schedules, long-term timetables, or even your study environment. This final day is all about self-care– making sure you are in tip-top mental and physical shape for the exam.

  • Watch cute videos
    Give yourself a mental break so that your mind is relaxed and ready to focus on test day. A great way to do this is through short, fun videos that make you laugh and smile. Choose videos that can be understood with little or no thought. In the graphic below, we’ve recommended some search keywords for cute, fun videos that will soothe your brain before you go in to take the GMAT.
  • Eat protein
    Amino acids fuel brain activity and boost GMAT performance. These chemicals are used to transmit messages and generate thoughts. Amino acids are found primarily in protein-rich foods. So eat a hearty-protein-filled meal before the test. In the picture below, I’ll give you a list of high-protein foods you can eat before the exam.
  • Make a checklist
    There are certain things you need to know, wear, pack, and do as you prepare to go to the test center and take your GMAT. Make a checklist of those things so that you can ensure everything is in order before the test. This will not only increase your chances of being prepared, but wit will also minimize the stress of making sure you’re prepared. A list of things to put on your checklist appears in the life hacks infographic below.
  • Do NOT cram
    You should do little or no studying on the day before you take the GMAT. This may sound strange or counter-intuitive, but it’s true. For one thing, good effective study takes weeks or months. One extra day of cramming for the GMAT won’t make much difference. For another, spending the entire day before the exam studying is stressful. It’ll wear you out, and you may be so stressed at the end of the day that you don’t sleep well. A day of intense studying isn’t conducive of the kind of self-care you need right before you take the test.
  • Sleep well
    Lack of sleep is one of the most common reasons that people do badly on the GMAT, even if they studied well and got good scores on their practice tests. Make sure you get in at least seven consecutive hours of sleep the night before the test. Eight hours of uninterrupted sleep may be better.
  • Use an alarm clock
    This may seem like a no-brainer. Of course you’ll need an alarm clock if you need to get up at a specific time and go take the GMAT. But here’s the thing: alarm clocks fail. You’ll want to have as many failsafes as you can. You need to guarantee that your alarm clock goes off, you hear it, and you wake up in a timely fashion on test day. See the following graphic for tips on rigging up a highly reliable alarm clock system.


GMAT Hacks 13-18: Things you can do on the test
Yes, there are even some simple hacks to do in the test center, and even during the testing session!

  • Brain dump
    A “brain dump” is when you quickly write down everything you’re thinking. This is a great way to both clear your mind of distractions and bring important information to the foreground of your consciousness. Right before you enter the testing room for the GMAT, spend a few minutes writing down your thoughts> Write down knowledge you have for the test and other things that may be weighing on your mind. You’ll be surprised at how much this can sharpen your focus and mentally energize you.
  • Chew gum

    The act of chewing gum is another way to sharpen your mind for the test. Studies show that the physical act of chewing is the real stimulant that enhances test performance. So the actual flavor or amount of sugar in the gum don’t matter. Just chew!
  • Predict the answers
    Before you look at all of your answer choices on the GMAT multiple choice questions, think up your own correct answer choice. This should be a true prediction– an educated guess rather than a blind guess. Base your prediction of the right answer choice on whatever you already know before looking at the actual choices. If your guess doesn’t show up in the answers, you’ll have to carefully check each option. But sometimes, your guess will turn out to be one of your choices. In that case, you can check your prediction first. This will lead you to the right answer much more quickly on a significant number of questions. Thinking of the answer before you check the choices also puts you in a  frame of mind to think more quickly and clearly.
  • Have good posture
    Keep your back straight as you enter the test center. Keep up that good posture while you sit for the test, and when you get up to take the occasional short break. Studies have shown that good body posture boosts your confidence. Improving your posture helps you be more alert too!
  • Get a (small) sugar rush
    Devouring sweets in large quantities isn’t healthy, and can hurt GMAT performance. But a little sugar can elevate your emotional mood and energize you. See the post for suggestions on sweets to eat right before you go into the test room, or on your breaks.
  • Breathe deeply
    The speed and depth of normal breathing is not the best pace of breath for optimal focus and concentration on. Health experts agree that deeper-than-normal breathing can boost your mental performance, enhancing calm and focus. See the image below for some additional deep-breathing tips from Magoosh.


Bibliography


Read a Life Hacks Blog or Two
As you can see from the bibliography, there are many websites out there when you can get good life hacking advice for your test prep. In addition to the sources above, I also recommend Lifehacker, a personal favorite lifehacks blog of mine. They have an article that gives more information on the Seinfeld X Technique, and their sections on motivation and goals have a lot of articles that are relevant to test prep.

The full image is below. Share using this embed code


</p><br /><br /><br /><br /><br /><br /><br /><br />

<p><a href=’magoosh.com/gre/2014/exam-lifehacks-infographic/’><img src=’https://magoosh.com/gre/files/2014/07/general_lifehack_800x3123.png’ alt=’Exam Lifehacks Infographic’ width=’540px’ border=’0′ /></a></p><br /><br /><br /><br /><br /><br /><br /><br />

<p>

Editor’s Note: This post was originally published in July 2014 and has been updated for freshness, accuracy, and comprehensiveness.

 

The post Exam Lifehacks Infographic appeared first on Magoosh GMAT Blog.
This Blog post was imported into the forum automatically. We hope you found it helpful. Please use the Kudos button if you did, or please PM/DM me if you found it disruptive and I will take care of it. -BB
Magoosh GMAT Instructor
Joined: 28 Dec 2011
Posts: 4450
Own Kudos [?]: 28569 [1]
Given Kudos: 130
Most Common GMAT Topics & Questions [#permalink]
1
Bookmarks
Expert Reply
FROM Magoosh Blog: Most Common GMAT Topics & Questions
Of course, a facile discussion of GMAT topics might say — the most common GMAT Verbal questions are Reading Comprehension, Sentence Correction, and Critical Reasoning — those three questions are the only questions you see on the Verbal section.  Similarly, all of the GMAT Quant questions are Problem Solving and Data Sufficiency.  



If you are not familiar with this general format, it’s vitally important to learn the variations and strategies for each of these five question types.  In fact, it’s important to be familiar with the format of the GMAT and how long the test is overall.

Most folks, though, know all this already.  Yes, the GMAT Quant section will be split between PS & DS questions, but given that format, what is the GMAT likely to ask?  That’s the real question.

Most Common GMAT Quant Questions


The same mix of topics show up on both the PS and the DS — those two GMAT math question types do not discriminate in any meaningful way as to mathematical concepts tested.

At Magoosh, we did an analysis of the frequency of GMAT Quant concepts in various official materials.  Here are the headlines.

The big granddaddy of GMAT quant questions is Number Properties, especially Properties of Integers — that one topic (actually, it’s a zoo of interrelated topics!) accounts for 10% – 15% of all math questions you see on the GMAT.

Another extremely frequent GMAT topic is percents (including interest & discounts) — which certainly makes sense for a test designed for folks going into business!

Other heavy hitters include (a) statistics, (b) two equations with two unknowns, (c) rate questions, (d) fractions, (e) exponents & powers, and (f) coordinate geometry.  Together, those are the most common GMAT Quant questions.

A couple more quick facts: of course, you should know that there is no calculator on the GMAT Quantitative section.  This means, regardless of topic, you need to practice your mental math skills, including estimation, divisibility tricks, and the doubling & halving trick.  Those are some of the most common skills you will need when answering GMAT Quantitative questions. It’s also important to know how to study for GMAT math.

Most Common GMAT Verbal Questions


Here, the most common GMAT topics vary by question format, so I will discuss the three formats separately.

Reading Comprehension: by far, the most common GMAT RC question is the “main idea/ author’s purpose” question.  The most important skill is being able to distill the main purpose both the whole passage and of any paragraph in the passage.  Another very important RC skill is knowing how to infer from the passage what is not explicitly stated.

Critical Reasoning: the three most common GMAT CR questions are the “weaken the argument” question, the “strengthen the argument” question, and the “find the assumption” question.   Those three are well over 50% of the CR section right there.  One crucial skill on the CR section is simply learning to distinguish the CR question types.

Sentence Correction: The two biggest categories for GMAT SC questions are “Logical Predication” and “Rhetorical Construction.”  “Logical Predication” has to do with the logical implications of the word order — misplaced modifiers, and the subtle shift in meaning we get between, say,  “not almost a hundred” vs. “almost not a hundred“.  Not completely different, “Rhetorical Construction” is about language that is direct, clear, and powerful, avoiding anything awkward or ambiguous or too wordy.  Together, these two are concerned with how well-crafted a sentence is.  Yes, you have to know correct grammar to craft a sentence artfully, but the focus is really at a higher level than the grammar per se.  Good grammar is the “clay”, the raw material, from which a cogent sentence is skillfully constructed.

Another very common GMAT SC question type concerns parallelism, another “higher level” concept.  Virtually any grammatical forms can be put into parallel, but the GMAT of course loves putting long complicated forms in parallel, such as participial phrase, gerunds, and infinitive phases.  The GMAT SC also loves comparisons, a special case of parallelism.

Finally, another common GMAT SC question type concerns idioms.  Yes, Virginia, the GMAT does ask about idioms.  Here, we don’t mean “idioms” in the sense of colorful metaphorical expressions (“scarcer than hen’s teeth“, “up the river“, “in a pickle“), but rather, the unique combinations of words that “sound right” — matching the right preposition to a verb, or knowing which verbs require infinitives, or phrasing a comparison the right way, etc.

Common GMAT Topics Summary
Below is a quick summary of some of the most common questions people ask about the GMAT.

Any further questions?  Let us know in the comments sections below!  🙂

 

Editor’s Note: This post was originally published in April 2013 and has been updated for freshness, accuracy, and comprehensiveness.

The post Most Common GMAT Topics & Questions appeared first on Magoosh GMAT Blog.
This Blog post was imported into the forum automatically. We hope you found it helpful. Please use the Kudos button if you did, or please PM/DM me if you found it disruptive and I will take care of it. -BB
Magoosh GMAT Instructor
Joined: 28 Dec 2011
Posts: 4450
Own Kudos [?]: 28569 [1]
Given Kudos: 130
MBA Admissions Advice for Career Changers [#permalink]
1
Bookmarks
Expert Reply
FROM Magoosh Blog: MBA Admissions Advice for Career Changers
If you are applying to an MBA program to pursue a career change, your application will carry an additional weight not shared by other applicants. You will have to show that you understand the new industry and/or role you’re targeting in practical terms. You will have to go beyond showing your real interest and make a rational case for your career goals. You will need to show that you comprehend your chosen field or job “from the inside.” Not accomplishing this objective will undercut the sincerity of your goals.

The goals essay or statement is where you show this understanding. You will need to describe how you plan to reach your goals, illustrating the steps you will take.

Here are 7 tips for using your goals essay to show that you know precisely what you are doing in changing careers.

1. Tell the story of your motivation and your growing understanding of your intended business or role.
This will make the change appear to be a natural progression in your professional development. Describe how your interest grew, specifying some of the more revealing aspects.

2. Show that you are cognizant of the day-to-day actuality of your chosen field/role.
While researching your new interest, you may have interviewed professionals in the field, taken classes or read publications about the industry. Use your essay to show your active pursuit of information to more readily understand the field and the awareness you gained.

3. Honestly focus on trials and impediments you expect to encounter as a newcomer to the field.
Explain how you plan to handle these difficulties. This will show additional knowledge of the field or job as well as your maturity, proving to the adcom that you are able to make the change.

4. Describe how your current job has enhanced your skills and helped you gain knowledge that you will be able to transfer to your new career.
Giving specific examples of skills or knowledge relevant to your new field shows both your preparation for the position and your understanding of its requirements.

5. Demonstrate how your “different” experience will allow you to make a unique contribution to your new role.
Outline these benefits, along with real-life examples.

6. Clarify something you wish to accomplish or contribute through your career, and the impact you want to have.
When you clearly state a vision for your goal, the adcoms will be excited about your vision of the future.

7. Use the “why this school” part of the essay effectively.
The things that you learn during your MBA will bridge the gap between where you are today and where you want to be in the future. Use this section to name things you will need to learn for your new career. This will underscore the fact that you understand the actuality and needs of that new career. Use specific points to show how this program will act as the bridge.

You may face unique obstacles in your MBA application as a career changer. Download our admissions guide, Why MBA, for help in identifying your MBA goals and organizing them in an essay that will get you accepted.

The post MBA Admissions Advice for Career Changers appeared first on Magoosh GMAT Blog.
This Blog post was imported into the forum automatically. We hope you found it helpful. Please use the Kudos button if you did, or please PM/DM me if you found it disruptive and I will take care of it. -BB
Magoosh GMAT Instructor
Joined: 28 Dec 2011
Posts: 4450
Own Kudos [?]: 28569 [0]
Given Kudos: 130
GMAT Tuesday: OG Reading Comprehension #1 [#permalink]
Expert Reply
FROM Magoosh Blog: GMAT Tuesday: OG Reading Comprehension #1
Welcome! We are looking at the first question in the reading comprehension section of the Official Guide to the GMAT. This is the second video in a series of 5. In this video, I cover how to approach detail questions, the common wrong answers in detail questions, and of course, how to arrive at the right answer.



Video Transcript
Hello, welcome to GMAT Tuesdays. My name is Kevin and I’m here from Magoosh. I’m sure you guys all know about Pokemon Go, all the kids are playing it. I was like, man, I can play Pokemon Go. Look at this Mewtwo Pokemon I caught. They’re super rare, isn’t that awesome?

Let’s dive into reading comprehension. In the official guide to the GMAT, we’re looking at question number one, the very first one. We’re in this book. There are newer official guides, but you’ll find this question in there as well. The previous video we did was about the passage. So now, we’re diving into question number one.

Passage Review: GMAT Official Guide Reading Comprehension Question #1
So, one thing to review before we dive into the question is what we discussed in the previous video. We went over the main idea, which is there’s two theories about fish schooling. We talked about the structure, first paragraph’s the intro, second paragraph talks about idea or theory number A, not number A, idea A.

And then paragraph number three is the critics of theory A and introduces theory B and then the fourth paragraph is all about theory B, the confusion effect. And then we talk about the tone, it is balanced-ish. In that the author is, basically, just presenting two theories of schooling.

Official Guide Reading Comp Question #1
So, question number one. I’m gonna just read straight from the passage, cause that’s easy. The question says, according to the passage, theory B states that which of the following is a factor that enables a schooling fish to escape predators? Great. So, this question is a detail question. I know that, because of the phrase “according to the passage.”

And when I know I’m dealing with a detail type question, I know there are certain things I need to do. This is my plan. The first one, attack the passage, which we all ready did. Second one, we’re gonna rephrase the question, which we can do right now. The question is basically just asking, what does theory B say about schooling and escaping predators?

Then I’m gonna do some research in the passage, I’m going to anticipate what an answer might sound like. So, sort of predict what the answer might be. And then finally, step five, go into the answer choices. Eliminate wrong answers until I arrive at the right answer. One other thing to remember is that they like to have common wrong answers with specific question types.

So with detail questions, we have three typical wrong answers. One, an inference, if an answer choice is making an inference, it’s wrong, cause we’re looking for a detail in the passage, which means it’ll be written in the passage somewhere.

[Two] A distortion of the passage where they take something in the passage and twist it, make it wrong just a little bit.

And then finally [three], when they introduce new information. Sometimes it’s related to what the topic is, but it isn’t in the passage at all. So, keep an eye out for the new information. Let’s dive into the answer choices.

Answer Choices
So answer choice A mentions compact groups, as a reason and this is a distortion of the passage. Compact groups that’s just what schooling is, but it doesn’t, that’s not what theory B is talking about. Theory B is explaining why compact groups would help the fish to avoid a predator. So this is a good example of a distortion, we can eliminate it.

Answer choice B is awesome. It is the right answer, cause they mentioned movement of the fish and I know I’m looking for theory B. So, I’m looking in the fourth paragraph for an answer. And if you look at lines 35 to 40, that’s where you find the description about the movement of the fish allows them to avoid being chomped on by predators.

Answer choice C, inability to detect. This is a distortion of the passage, because this is what theory A says. So, this is a perfect example of how they’re distorting the passage. We have a question that’s asking about the second theory and they provide an answer choice that is about the first theory. So they wanna make sure you’re paying attention and reading closely and know what they’re actually asking about, cause you would find them in the passage, but it’s not related to theory B, it’s related to theory A.

And answer choice D says that the fish can hide behind each other, which is an interesting concept, but not mentioned in the passage. We have no details and no information about fish hiding behind each other.

And then finally, answer choice E, that the fish can move with great speed. Maybe that might be a reason, but it’s not brought up in the passage. There’s nothing in the fourth or third paragraph where we learn about theory A or theory B, excuse me, where the fish move really fast and that helps them do avoid the predator. So, this is new information not included in the passage. So, easy to eliminate. Alright, all you GMAT studiers.

I hope this was helpful. If you have any questions, leave comments down below. And if you need any more help, head over to Magoosh where there’s a lot of friendly and fun people who can help you with their GMAT studies and help you capture Pokemon.

This is how you play Pokemon Go right? You just capture, capture.

The post GMAT Tuesday: OG Reading Comprehension #1 appeared first on Magoosh GMAT Blog.
This Blog post was imported into the forum automatically. We hope you found it helpful. Please use the Kudos button if you did, or please PM/DM me if you found it disruptive and I will take care of it. -BB
Magoosh GMAT Instructor
Joined: 28 Dec 2011
Posts: 4450
Own Kudos [?]: 28569 [0]
Given Kudos: 130
Grad School: Bigger Debt, Bigger Rewards [#permalink]
Expert Reply
FROM Magoosh Blog: Grad School: Bigger Debt, Bigger Rewards
We’ve all heard the horror stories about student loan debt by now. Seven out of 10 students take on debt to earn their bachelor’s degree, and the amounts that some borrowers end up owing can be intimidating.

Many students headed for graduate school probably think they already know the ropes when it comes to taking out student loans. But when you move on to grad school, the rules — about how much debt you can take on, the types of loans you’re eligible for, and how much it will cost you to pay them back — all change.

Not only do grad students often take on much more debt than undergraduates, but they’ll typically be paying higher interest rates. In many ways, grad school student loan debt is like regular student loan debt on steroids.

Of course, many graduate and professional degrees boost the job prospects and earnings power of their recipients considerably, making student loan payments more manageable.



Source: U.S. Bureau of Labor Statistics

The standard advice you’ll get from financial aid counselors is to round up all of the grants, scholarships, work study and help from your family that’s available to you before taking out loans to pay for school. If you have to borrow to pay for your education, the conventional wisdom is to max out on federal student loans before turning to private lenders.

But some federal student loans are a better deal than others. In today’s low interest rate environment, private student loans can be competitive with the pricier government loans — particularly when compared to federal PLUS loans for grad students. It’s important to keep in mind, though, that while rates on government loans are fixed for life, private lenders offer both fixed- and variable-rate loans. Variable-rate loans typically carry lower initial interest rates, but the rate can change over time.

Loans for grad students
Once you move on to graduate school, you’re no longer eligible for direct subsidized loans and the relatively low interest rates that you might remember from your undergraduate studies, regardless of your financial need. Rates on direct unsubsidized loans for graduate students are significantly higher — they’ll be 5.31 percent for new loans issued after July 1, 2016. And unlike subsidized loans, interest on unsubsidized loans starts accruing as soon as you take your loan out.

Loan typeBorrower typeInterest rate

Direct Unsubsidized LoansGraduate or professional5.31%

Direct PLUS LoansParents and grad students6.31%

Interest rates on federal direct loans for grad students. Source: U.S. Department of Education

 

The aggregate borrowing limit (undergrad loans + grad school loans) for direct subsidized and unsubsidized federal loans is $138,500. Medical school students and others earning certain health-related degrees can take out up to $224,000 in federal direct loans. If you need to take out more than that to cover grad school costs, the government also offers PLUS loans.

PLUS loans
While you can take out all of the PLUS loans you need to cover school attendance costs, it will cost you more to pay them back. Interest rates on PLUS loans issued from July 1, 2016 through June 30, 2017 will be 6.31 percent. On top of that, PLUS loans carry a costly 4.3 percent up-front disbursement fee. That fee is taken out of the loan before you even see the money. For loans paid back on a standard 10-year repayment plan, the disbursement fee has roughly the same effect as adding one percentage point to the annual percentage rate (APR).

PLUS loans are also not subsidized while you are studying. If you make no payments for two years while enrolled in grad school, it will cost you more than $42,000 to repay $30,000 in PLUS loans at today’s rates, with 10 years of payments at $353 a month. That’s in part because interest will accrue on your loans while you’re still in school, adding $1,159 to the balance before you’ve even started making payments. All told, you’d rack up about $12,000 in interest in the course of paying off this loan (you can use FinAid.org’s student loan calculator to run your own numbers).

Private student loans
Before taking out a federal PLUS loan for graduate school, it’s worth taking a look at private student loan lenders.

When shopping for private student loans, keep in mind that government loans are “one size fits all” — everybody who takes the same type of loan out at a given time pays the same rate, and those rates are fixed for life. Private student loans offer more options (and sometimes more risk!) when it comes to rates and terms.

With private student loans, a good rule of thumb is that the longer the loan term — the number of years you’ll take to repay the loan — the higher the interest rate, and the lower the monthly payment. The interest rate also depends on the borrower or cosigner’s credit risk, and whether you’d rather have a fixed-rate or variable-rate loan.

Pick a variable-rate private student loan, and you’ll start out with a better interest rate than you’d get on a fixed-rate private loan with the same repayment term. But there’s some uncertainty involved with variable-rate loans — they’re tied to indexes like the prime rate or LIBOR and can move up and down over time.


In the chart above, gray bands indicate periods of recession, when policymakers often lower rates. If you had to guess, which way would you say rates are headed next? Source: Federal Reserve Bank of St. Louis.

 

If you’d rather have the certainty of a fixed-rate student loan, most private lenders offer those, too. You’ll pass up the chance to start out making lower monthly payments but if interest rates go up, your monthly payments will remain unchanged.

Repayment plans
Whether you’re relying on government or private student loans, the time to start thinking about repaying them is when you take your loans out, not after you graduate.

When choosing between private and government student loans, keep in mind that there’s more to consider than the interest rate. Private student loans don’t offer some of the borrower benefits packaged with federal loans, like access to income-driven repayment (IDR) plans — where your monthly payment is dependent on how much you earn– and the potential for loan forgiveness after 10, 20, or 25 years of payments.

IDR plans can help graduates with modest earnings pay off big loan balances. So if you know you’ll be taking on big student loan debt to earn a degree that may not command a lucrative salary, a government IDR plan might be a good safety net.

But IDR plans aren’t the best solution for everyone. If you won’t qualify for loan forgiveness, stretching out your payments over a longer period will typically increase the total amount repaid. If you do qualify for loan forgiveness, you may face a large tax bill.

Many private lenders are adopting borrower-friendly features like in-school grace periods and optional deferment. Remember that interest will accrue on your loans during these periods, just as it does on unsubsidized federal direct loans and PLUS loans. Private lenders typically offer several repayment plans, including the option to keep a lid on costs by paying the interest on your loans while you’re still in school.

Shop around
If you are considering a private loan, it’s a good idea to comparison shop since private lenders compete for your business. Because the rates and terms offered by private lenders depend on the borrower or cosigner’s creditworthiness, you need to do a little homework to find the actual rates you’ll qualify for.

The student loan marketplace Credible.com offers an online comparison tool that let’s you find the actual rates you could qualify for with multiple lenders, without sharing your personal information during the shopping stage or affecting your credit score.

As an example of rates offered by lenders on the platform as of July 1, fixed-rate student loans from Citizens Bank were available at rates as low as 3.74 percent, with variable-rate loans starting at 2.45 percent. Keep in mind that not all private student loan lenders are on the platform. Your school’s financial aid office may also maintain a list of private lenders.

Refinancing
Whether you rely on federal or private student loans to earn your graduate degree, you can explore refinancing some or all of your student loans with a private lender after graduation. The government might also offer student loan refinancing in the future. As things stand, if you choose to refinance a federal loan with a private lender, you will lose some federal protections and access to federal repayment plans.

Your income and credit history may qualify you for lower rates. With interest rates hovering near historic lows, the potential savings are significant, particularly for those with higher-interest PLUS loans or grad school debt.

 

 

 

 

We’re test prep experts here Magoosh, not financial aid experts. The views expressed here belong to Credible. We encourage all of our students to do sound research on the Federal Student Aid website when making important financial decisions!

The post Grad School: Bigger Debt, Bigger Rewards appeared first on Magoosh GMAT Blog.
This Blog post was imported into the forum automatically. We hope you found it helpful. Please use the Kudos button if you did, or please PM/DM me if you found it disruptive and I will take care of it. -BB
Magoosh GMAT Instructor
Joined: 28 Dec 2011
Posts: 4450
Own Kudos [?]: 28569 [1]
Given Kudos: 130
How to Overcome a Low GPA When Applying to MBA Programs [#permalink]
1
Bookmarks
Expert Reply
FROM Magoosh Blog: How to Overcome a Low GPA When Applying to MBA Programs
Applying to business school is a stressful process. And having a low undergraduate GPA does not help. Of course building a business school list that encompasses reach, fit, and safety schools is always a good solution. But many people applying to MBA programs have their eye on top programs, which accept only the most qualified students.

But a lower than average GPA does not mean you can’t get into HBS or the GSB or Wharton. Here are a five ways you can overcome a low GPA when applying to MBA programs:

1. A Good GMAT Score
One of the best – and certainly one of the most obvious – ways to overcome a bad GPA when applying to business school is to get a good GMAT score. A strong standardized test score is another indicator that you will be able to handle the material at top MBA programs. It can help you combat the numbers game in addmissions because your hard stats (your GPA and your GMAT score) are the initial impression you will make on an admissions reader.

While, a good GMAT score can begin to counterbalance a low GPA, your GPA typically carries more weight than your GMAT. John Lyon, a former admissions officer from Wharton said, “Your GPA reflects your performance over time, while your GMAT reflects how well you test on a given day. But a low GPA in hard sciences or math is more acceptable than in social sciences or humanities.”

That being said, your scores are not everything. Particularly with business schools, the admissions officers take a holistic approach in looking at your application. Your essays, letters of recommendation, and outside activities play a large role in admissions.

2. Write an Addendum Explaining Your GPA
Do this only if you have legitimate extenuating circumstances that led to a bad GPA.

Did your GPA drop drastically the semester that a parent got sick? Was there an upward trajectory once you overcame a hard transition freshman year?

Having a low GPA when applying to MBA programs is a common problem. Explaining these situations is worth the time and energy it takes to write a good addendum. But, if your grades dropped every semester of college because you were sleeping in instead of attending lecture, it is best to leave it alone. Do not use the addendum to make excuses.

3. Extracurricular Activities
MBA programs are looking for candidates who excel in the classroom, but who are also involved outside of it. Building a cohesive list of business school extracurriculars activities and volunteer experience is a good way to combat a bad GPA.

Say you were a Computer Science major now working in Business Development at a tech job, and you’re applying to business school to gain more entrepreneurial skills to start your own company. Maybe you were the president of the robotics club at your undergrad university. And maybe you start a foundation that provides coding classes to low-income communities in your area. These activities all support a specific application persona or central theme. If you can build out a strong and cohesive set of activities that tell a unique story and display leadership, your application is going to be more compelling.

4. Work experience
This one’s important. What has been your work trajectory over the past few years: Are you a manager? Did you lead a development team? Were you the founding member of a company-changing project?

Showing experience and leadership in the workplace is extremely important in applying to MBA’s (unless you’re applying to Harvard’s 2+2 program or another equivalent straight out of college). Business schools want students who will be leaders and change-makers in their field. If you show that you have already started to move in that direction in your early career, your application will be more compelling.

5. An answer to: “Why an MBA now?”
A good, compelling answer to this question is extremely important. If your answer is along the lines of “because you will make a lot more money with an MBA,” then you should rethink your decision. Rather, if you’ve been climbing the ranks of a large firm and want to take your managerial skills to the nonprofit or startup world, you’re most likely going to tell a more compelling story.

MBA programs want to see that you will use your schooling to grow your skills and make a difference in the business world, not to make money in the business world.

Having a low GPA when applying to MBA programs is not necessarily the end of the world in terms of your application. There are always ways to combat the numbers, and the admissions office will take everything into consideration – from you GPA to your letters of recommendation. So study for the GMAT, build out your resume, and put together a compelling story in your application.

The post How to Overcome a Low GPA When Applying to MBA Programs appeared first on Magoosh GMAT Blog.
This Blog post was imported into the forum automatically. We hope you found it helpful. Please use the Kudos button if you did, or please PM/DM me if you found it disruptive and I will take care of it. -BB
Magoosh GMAT Instructor
Joined: 28 Dec 2011
Posts: 4450
Own Kudos [?]: 28569 [0]
Given Kudos: 130
How to Study for GMAT Verbal [#permalink]
Expert Reply
FROM Magoosh Blog: How to Study for GMAT Verbal
Many GMAT Verbal improvement tips are focused on the nitty-gritty: nail your idioms, learn to deconstruct the elements of the Critical Reasoning argument, know what constitutes a wrong answer on reading comprehension—and what constitutes a right answer. Those are without a doubt important, and if you haven’t learned them yet that should be your next order of business. But this post is going to focus on an oft neglected but equally important area for success on the GMAT Verbal Section: your reading brain.

Your what? Yes, I know. I should explain first. The GMAT Verbal is not your average English. It is sophisticated and academically inflected. As such, it is not easily digestible. Unless, that is, you make it a habit to read this kind of writing—and I don’t just mean reading GMAT passages (that would be a most unwelcome form of prep!) By becoming a stronger reader, your reading brain will grow. Taking the verbal GMAT will not be nearly as difficult or at least as mentally draining. Sure, you’ll still need to know those nitty-gritty details upon which a question hinges. But with a strong reading brain, you’ll have a better shot at picking up on these things in the first place.



Before the GMAT Verbal Test: Read
Do not think of GMAT prep as an isolated activity, done solely over the familiar spread of books. To truly prepare yourself for the GMAT Verbal Section you should immerse yourself in words – books (fiction and non-fiction), magazines (long and short form), and newspapers (the ones with national circulation).

If this sounds superfluous, remember that developing an ear for proper English comes far more naturally than imbibing a list of idioms. Of course not all English is equal. Be sure to avoid the colloquial form as much as possible, and instead choose sources venerated for their elevated style: The New York Times vs. USA Today, The New Yorker vs. The National Enquirer.

It is also important to read what interests you. Nowhere is this more true than with books, both fiction and non-fiction. To slog through a book, even a revered classic, is going to make you disdain this process. On the other hand, a page-turner doesn’t have to be John Grisham, or even pulpier tomes. Have a look at the Modern Library 100’s Top Books. There is something for every taste here (and with Amazon you can click “Look Inside” to see if the books suits you).

And for those who live near bookstores – assuming they have not all gone the way of Tower Records – scan the book displays. As long as the topic is relatively sophisticated, the book is sure to have correct, idiomatic (read: GMAT) language.

Plan on reading a total of 50-pages a day from a variety of these sources. Doing so will fine tune your brain and make Reading Comprehension far less formidable. Finally, the sooner you do this the more time you’ll give yourself to grow your “reading brain”.

Think in Your Own Words, Not in GMAT Verbal Vocabulary
Now that we’ve covered the reading brain, I want to focus on other aspects of the GMAT Verbal Section that are still holistic and are often glossed over in most strategy guides.

Yet, it is still connected to the idea of being a stronger reader, one adept at navigating the treacherous verbiage of the GMAT. Your ability to process words will increase as will your ability to make sense of what you read.

This latter skill is crucial for Critical Reasoning and Reading Comprehension. When you read a question, you will want to be able to anticipate the answer. That is you want to verbalize a rough answer. That way you control the question – instead of the letting the answer choices control (and corrupt) you.

Of course this anticipation of the correct answer also carries over to Sentence Correction. By developing your ear for proper English, you should be able to identify what is wrong with the underlined section.

While reading outside is important, ultimately your score will depend on the GMAT sources you use. Make sure you learn the fundamentals of the verbal section. Manhattan GMAT and Magoosh do a very good job of breaking down the test so you can understand the principles behind the question, instead of relying on clever guessing strategies.

Focus on One Area of the GMAT Verbal Section
Okay, I promised to talk about holistic tips to GMAT Verbal success. This next tip falls under strategies, yet it is still holistic since it doesn’t deal with any one section. Rather, it deals with how you approach studying the different sections.

When first start off, I found that it is good to spend a day or two just focusing on one area. That way you are able to make progress, versus jumping from prepositional phrases (Sentence Correction) to paradox question types (Critical Reasoning).

So do not try to do much at once. Pick a certain area in Critical Reasoning and stick to it for a few days. For instance, practice inference questions to the point where you are comfortable with anything up until the 650-level. You can carry this approach over to Sentence Correction and focus in just one area, say Parallelism. Once you have strong grasp, move on to another concept, but review every few days.

Once you’ve made clear progress in one area, then it makes sense to do daily practice sets in which you mix up verbal question types. After all, this is exactly what you’ll face test day.

Review What You Read
Speaking of review, sometimes it is tempting to just push forward, constantly doing new questions and tackling new concepts. While noble, this approach is scattered and will not allow concepts to really sink in. Make sure you review what you’ve learned. So don’t think of a question as one you’ve already done. For you to truly be able to move on from a question, you should be able to effortlessly articulate why the wrong answers are wrong and right answer is right.

Takeaway
By combining reading with smart GMAT prep, you can boost your verbal score quickly, whether you’ve been speaking English since two or consider English as a third language.

Would you like some more GMAT Verbal Tips? Check out these posts from the Magoosh GMAT Blog:

 

Editor’s Note: This post was originally published in January 2012 and has been updated for freshness, accuracy, and comprehensiveness.

 

The post How to Study for GMAT Verbal appeared first on Magoosh GMAT Blog.
This Blog post was imported into the forum automatically. We hope you found it helpful. Please use the Kudos button if you did, or please PM/DM me if you found it disruptive and I will take care of it. -BB
Magoosh GMAT Instructor
Joined: 28 Dec 2011
Posts: 4450
Own Kudos [?]: 28569 [0]
Given Kudos: 130
Introduction to GMAT Critical Reasoning [#permalink]
Expert Reply
FROM Magoosh Blog: Introduction to GMAT Critical Reasoning
One question on the GMAT Verbal Section is Critical Reasoning.  On this question, the prompt presents some sort of argument, and then asks you, in one way or another, to analyze the argument—for example, by strengthening it, weakening it, finding its underlying assumption, etc.   The argument prompt is typically less than 100 words, much shorter than a Reading Comprehension passage, and there’s always only a single question on the Critical Reasoning argument.  Critical Reasoning comprises roughly 1/3 of the Verbal Section, about 13 Critical Reasoning questions of the total of 41 Verbal Questions.

After some discussion, this post features GMAT Critical Reasoning practice questions below.



Why does the GMAT ask Critical Reasoning?
You are preparing for the GMAT, which ostensibly means you are planning on attending business school, and this in turn suggests that you are anticipating a management career in some aspect of the business world.  The entire business world runs on buying and selling: even if you are not a salesperson yourself, the success of your business, in a sense the raison d’etre of the business, depends on the money it will make from sales.

Well, in its essence, every sale is an argument.  If I want to sell you sometime, I have to present a case in some form to convince you to buy it.   If I make a wonderfully cogent argument, I may well generate the sale.  If my argument is faulty, and I repeat this pattern, that can only mean bad things for the long-term financial well-being of my business.

Every sales is an argument, but that’s just where the arguments in the business world start.  How does you company decide whether one strategy or policy is better than another? What motivates your company to buy from a supplier?  What motivates your customers to remain buying from you? What concerns does your insurance company have about your company? What concerns do your company’s investors have?  All of these important points, and many more, will be settled by arguments.

A typical manager has to deal with arguments from all quarter all day.  An effective manager has to be skilled at deciding: how would I strengthen or weaken this argument?  what is the assumption of this argument?  what further evidence would I need to evaluate this argument?  In other words, a real-life manager needs to apply all the skills required for Critical Reasoning on the GMAT.  Arguments are very important in business, and the skill of evaluating arguments is one that every manager should cultivate.  That’s precisely why business schools want you to bone up on it, which is why the GMAT asks about it in Critical Reasoning questions.

The 8 Types of CR Questions
Step one of the general strategy for GMAT Critical Reasoning is: read the question before reading the argument.  Know which type of question you are going to have to answer, and read the argument with that question in mind.  The eight broad categories of GMAT Critical Reasoning questions are

1) weaken the argument/find the flaw in the argument

2) strengthen the argument

3) find the assumption (know the Negation Test)

4) draw inference/conclusion

5) structure of the argument, including boldface structure questions and dialogue structure questions

6) paradox

7) evaluate the conclusion

8 ) complete the argument

Types #1-4 account for approximately 75% of all GMAT CR questions.  You can find out more about each one of these types at those linked blog articles and in the CR video series in the Magoosh product.  The basic idea is: when you know what you need to do, you will be reading the argument with that in mind.

Know What You’re Looking For
In all Critical Reasoning questions, the GMAT gives one correct answer and four tempting and potentially confusing statements for the other choices.  Folks who read the argument & question, and then wander aimlessly into the answer choices without any further thought are asking to be perplexed, and chances are, they spend much longer than necessary on many Critical Reasoning questions.

Go into the question with an idea of what you seek.  For types #1-3, the best thing to do is to find the assumption of the argument—-reaffirming or undercutting the assumption of an argument is the most powerful way to strengthen or weaken it. Finding the assumption may also be helpful in find the flaw of the argument (if the flaw is a faulty assumption).

For the other question types, you will be less able to predict what the answer will be, but still formulating the task in your own words will help you.  In your own words, what is the structure of the argument? What is the paradox that need to be resolved? What kind of information would be required to evaluate the conclusion? etc.  The clearer you can be on what type of information or argument will satisfy the question, the quicker you will be in finding it.

If you can simply integrate these strategies, you will find you are able to crack GMAT Critical Reasoning questions faster and more accurately.

 

 

GMAT Critical Reasoning Practice Questions
Here is a collection of ten Critical Reasoning practice questions. 

 

1) In order to combat Carville’s rampant homeless problem, Mayor Bloomfield recently proposed a ban on sleeping outdoors in the city’s many parks. He claims that such a measure will force the homeless to either leave Carville or to find means other than sleeping in public parks.

Which of the following, if true, suggests that Mayor Bloomfield’s plan will be successful?

(A) Until the ban, the city’s many homeless shelters were at less than fifty percent occupancy.

(B) Many homeless tend to congregate underneath Carville’s numerous overpasses.

(C) Adjacent cities have even tougher measures on the homeless sleeping outdoors.

(D) The percent of Carville’s population that has been homeless has been slowly decreasing in the last five years.

(E) Mayor Jonesmith, Mayor Bloomfield’s predecessor, had been far more tolerant towards the city’s homeless population.

 

2) Megalimpet is a nationwide owner of office space.  They have major office buildings in the downtowns of several cities in the 48 lower states, and rent this space to individual companies.  Megalimpet office spaces vary from small office to large suites, and every space has custom-designed wall-to-wall carpeting.  The carpet in several Megalimpet facilities needed replacing.  The winning bid for the nationwide carpet replacement was submitted by Bathyderm Carpet Company (BCC).  The bid contract involves all delivery costs, all installation, and any ongoing maintenance and upkeep while the carpet is under the three-year warranty.  Both BCC executives and independent consultants they hired felt BCC would be able to perform all these services for far less than their bid price; these circumstances would allow BCC to reap a considerable profit.

Which of the following, if true, most calls in question the argument that BCC will make a large profit from this contract with Megalimpet?

(A) All the carpets will have to be transported by train from BCC factory in Louisville, KY, to Megalimpet’s locations from coast to coast.

(B) BCC has already supplied carpets to a number of restaurant chains, and some of those spaces are as large as Megalimpet’s largest office spaces.

(C) The carpet installation teams will have to cut different sizes of the carpets for the different size office suites in the Megalimpet buildings.

(D) The material in BCC carpets degrades rapidly when it comes into contact with standard toner, found in most laser printers and photocopiers; the degraded sections are unsightly and smell bad, so they often need to be replaced.

(E) The next competing bid after BCC’s was 50% higher than BCC’s bid

 

3) A minor league baseball franchise experienced a drop in attendance this week after they suffered three losses by margins of ten runs or more last week.  Many spectators of those games wrote letters to the editors of the local sporting news, complaining of the poor play of the team in those three losses.  Nevertheless, the front office of this baseball franchise maintains that the team’s poor play in those three losses has nothing to do with this week’s decline in attendance.

Which of the following, if true, most strongly supports the position held by the front office of the baseball franchise?

(A) The spectators who wrote letters to the local sporting news were long-standing fans of this minor league baseball team.

(B) Many minor league baseball franchises attribute a drop in attendance to the quality of play of the team only after a string of losses.

(C) Other minor league teams in that region of the state reported a similar drop in attendance this week.

(D) This was not the first time this team suffered multiple lopsided losses in a single week, prompting similar letters to the local sporting news.

(E) This minor league team is over four hours from the closest major league team, so many of the minor league team’s fans do not often attend major league games.

 

4) In a few recent cases, some teenagers with advanced programming abilities used a new programming language, FANTOD, to hack into ETS and change their own SAT scores.  All of the teenagers convicted of this crime were highly skilled in programming FANTOD.  In light of these cases, some colleges have discounted the official SAT scores of applicants with a knowledge of FANTOD, and have required them to take special admission tests in supervised conditions on their own campuses.

Which of following conclusions can most properly be drawn from the information above?

(A) Most people who learn to program in FANTOD do so to commit some kind of hacking.

(B) Colleges should rely on their own admissions tests instead of the SATs

(C) The college admission process possibly places some students with knowledge of FANTOD at a disadvantage.

(D) Students who learn FANTOD tend to have much lower SAT scores than do their peers.

(E) Not all colleges requiring special admissions tests have administered these tests under supervised conditions.

 

5) In the twentieth century, the visual arts have embarked on major experimentation, from cubism to expressionism.  While tastes always vary, there are certainly some people who find beautiful objects of each of the art movements of the first half of the twentieth century.  In the latter half of the twentieth century, though, most works are so abstract or shocking that neither the critic nor the general public uses the word “beautiful” to describe them: indeed, sometimes late twentieth-century artists have, as one of their expressed goals, the creation of a work that no one could find beautiful.  Whatever these artists are creating may be intellectually engaging at some level, but it is no longer art.

Which of the following is an assumption that supports drawing the conclusion above from the reasons given for that conclusion?

(A) Art critics generally have a different appraisal of a work of art than does the general public.

(B) The meaning of any work of art is defined entirely by the ideas of the artist who created it.

(C) Beauty is a defining quality of art.

(D) All art movements of the latter half of the twentieth century are responses to the movements of the first half of the century.

(E) It is not possible for any work to be simultaneously beautiful and intellectually engaging.

 

6) The National Farm Administration (NFA) has been concerned over the last decade with the struggles of barley growers.

Plan: In an effort to support these barley growers, two years ago, the NFA began a program of sending them, each autumn, a free special mix of fertilizer and enzymes designed to multiply barley yield, to be applied the following spring during first growth.  This mix had been stunningly successful in multiplying the yield of barley in laboratory conditions.

Results: Most barley growers reported little change in their economic status over this two year period.

Further information: All barley growers received the shipments, and all used them.  Weather conditions have been fair to optimal for barley growth over the past two years.

In light of the further information, which of the following, if true, does most to explain the result that followed the implementation of the plan?

(A) During these two years, most of the barley growers reported using no other fertilizer besides the special mix sent by the government.

(B) The trucks that drove the special mix from the depot in Wisconsin to the individual farms sometime took as much as 4 or 5 days.

(C) Some of the enzymes in the special mix multiply the growth of a bacteria that feeds on the young barley plants.

(D) This program was implemented at a time when more than half of barley growers nationwide were reported barely breaking even in their yearly expenses.

(E) This was the second such NFA program to aid barley growers; the first one, 14 years ago, was started with high hopes, but did little to change their situation.

 

7) When, on a particular shopping trip, a consumer purchases an item which he previously had no intention of purchasing, this sale is called an “impulse purchase.”  The objects of impulse purchases are occasionally essential items (i.e. items that satisfy basic subsistence needs), but much more frequently are luxury or non-essential items.  Researchers have determined that, at the end of a shopping trip, a consumer is much more excited if she has bought a luxury item on an impulse purchase, than if she had made no impulse purchases.

If the information above is true, and if the researchers’ investigation was properly conducted, then which of the following must also be true?

(A) The impulse purchase of a luxury or non-essential item is more exciting than the impulse purchase of an essential need.

(B) A consumer who, for whatever reason, is not able to purchase an item she had planned to buy is necessarily disappointed.

(C) Consumers seeking a high level of excitement often make impulse purchases.

(D) The researcher had a reliable way to determine whether the consumer had planned to buy the luxury or non-essential item he purchased on that trip.

(E) The probability that a consumer makes an impulse purchase of an item decreases the price of the item increases.

 

8) Over the past ten years, the population of Dismaston has grown five times as large as it was.  During this time, the average income in the city has risen substantially, and a tremendous amount of capital has flowed into city.  An independent audit found that, somewhat surprisingly, the number of violent felonies reported per year is now lower than it was ten years ago.

Each of the following statements below, if true, would explain the somewhat surprising finding EXCEPT:

(A) White collar crimes, which are almost always non-violent, tend to replace street-crimes during times of prosperity.

(B) The police now have a computerized filing system, so that it is almost impossible for a violent crime to be unrecorded.

(C) During this time, the state considerably lengthened felony convicts’ waiting period for parole.

(D) The police force has expanded in number and is equipped with the latest crime detection technology.

(E) The city is now much better lit at night, and security cameras protect a large number of public venues.

 

9)  Archeologists have discovered three sites showing conclusive evidence for the mastery of fire in Tanzania, from a period slightly after the time that Homo habilis was present in Africa.  These sites clearly were founded by Homo erectus, the descendent species of Homo habilis that migrated north, out of Africa and into Asia. Homo erectus was known to have mastered fire, from ample evidence at sites in Asia.  There is no reason to attribute mastery of fire to Homo ergaster, the descendent species of Homo habilis that remained in Africa.

Which of the following is an assumption on which the argument depends?

(A) Before their migration, Homo erectus occupied African territory as far south as Tanzania.

(B) The strain of migration provided the selective pressure motivating Homo erectus‘ mastery  of fire.

(C) Homo ergaster would not have derived as much benefit from the mastery of fire as did Homo erectus.

(D) Homo ergaster inherited all cultural knowledge from Homo habilis, a species that did not have mastery of fire.

(E) Homo ergaster did not occupy regions as far south as Tanzania until well after the time of these three sites.

 

10) Five years ago, the town of Bayside, in the Katonic River Valley, had catastrophic flooding one spring, and consequently, most insurers now refuse to write flood insurance for houses in Bayside.  The town of Dryadia, in the Phemptic River Valley, is much like Bayside in its proximity to a similar river at an almost identical point in the river valley.   We can conclude that the only reason the same insurers do not write flood insurance for houses in Dryadia either is its similarity to Bayside in terms of where it is situated in the river valley.

Which of the following, if true, would most seriously undermine the argument?

(A) A small number of independent insurers will write flood insurance for at least some houses in each of the two towns.

(B) It is hard for an homeowner to buy flood insurance if a large proportion of other houses in the same town have been flooded in recent years.

(C) In many other towns in the Katonic River Valley, it is hard for home-owners to buy flood insurance.

(D) The town of Dryadia has some flooding most springs.

(E) Flooding from spring surges in rivers is only one of the ways in which a home can become flooded.





Practice Question Explanations
For the first six GMAT Critical Reasoning practice question, follow the link.  Re-enter your answer and submit it, and the subsequent page will have a full video explanation.

1) https://gmat.magoosh.com/questions/3413

2) https://gmat.magoosh.com/questions/3118

3) https://gmat.magoosh.com/questions/3110

4) https://gmat.magoosh.com/questions/5405

5) https://gmat.magoosh.com/questions/3137

6) https://gmat.magoosh.com/questions/3128

 

For the last four practice questions, the explanations are given in the text here.

 

7) The credited answer is choice (D).  If the researcher was able to conclude anything about how an impulse purchase made someone feel, then the researcher first had to know that it was an impulse purchase, that is, that the purchase was not planned.  If the researcher had no way to determine whether a purchase was planned or unplanned, then the researcher would have no way of determining which purchases were impulse purchases.

We know the consumer find the impulse purchase of a luxury item more exciting than the planned purchases.  We don’t necessarily know how exciting the impulse purchase of an essential need is—maybe it’s less exciting than the impulse purchase of a luxury item, or maybe it’s just as exciting.  We suspect from real life that this may be true, but we cannot determine this from information in the prompt, so it can’t be the answer to a “must be true question.” Thus, choice (A) is incorrect.

We only know about the excitement brought about by an impulse purchase of a luxury item, but we have no information about what happens if a purchase is planned but not made. Choice (B) inappropriate extends the pattern into situations the prompt doesn’t cover at all.  Choice (B) is incorrect.

We know that the impulse purchase of a luxury item is exciting, but we don’t know whether this is sufficient inducement for a person seeking excitement to make this kind of purchase frequently.  The expense, for example, might be a mitigating factor.  We can conclude nothing for certain about this, so choice (C) is incorrect.

This is a tempting one—we certainly might suspect that the luxury items of higher price would be bought as impulse purchases less frequently.  We might suspect this, but notice that the prompt says nothing about high price vs. low price items.   This answer choice invites us to bring in irrelevant outside knowledge, so, like (A), it can’t be the answer to a “must be true question.”  Choice (E) is incorrect.

 

8) This is an EXCEPT question.  Four of the answer will be perfectly valid explanations, and these will be incorrect.  One of the answers will not be good explanation—either it will be irrelevant, or it may even suggest a rise instead of a decline; this oddball choice will be the correct answer.

The credited answer is (B).  The new filing system, in essence, never misses the report of a violent crime.  This at least implies that perhaps the previous filing system missed some violent crimes on occasion—for whatever reason, some violent crimes that took place slipped through the cracks and failed to be reported.  Well, if we were not reporting everything before, and are reporting everything now, if anything this might suggest an increase in the number of reported violent crimes.  It most certainly would not, by itself, explain a decrease.  This is not in any way a good explanation, so this the correct answer.

We know, over the past decade, “the average income in the city has risen substantially” and “a tremendous amount of capital has flowed into city,” both of which indicate conditions of prosperity.  Therefore, according to choice (A), white-collar crimes would increase, and street-crimes would decrease, with a concomitant drop in violent crimes.  Choice (A) is a valid explanation, so it’s an incorrect answer.

If the state kept convicts in jail longer, that would mean fewer of them would be back out on the streets committing felonies, most of which are violent.  Therefore, it would lead to a drop in the number of violent crimes.  Choice (C) is a valid explanation, so it’s an incorrect answer.

Better police and better crime detection means more arrests and fewer violent crimes.  Therefore, it would lead to a drop in the number of violent crimes.  Choice (D) is a valid explanation, so it’s an incorrect answer.

Better lighting at night and security cameras have some effect in reducing crime.   Choice (E) is a valid explanation, so it’s an incorrect answer.

 

9) Remains of prehistoric fire were found in Tanzania.  The author says that Homo erectus made these fires, and that there’s no reason to assume Homo ergaster did.  What is a necessary assumption?

The credited answer is choice (A).  Homo erectus had to be as far south as Tanzania—if they were not, there would be no way they could have made those fires there, which would seem to indicate that Homo ergaster made them after all.  Negating this statement devastates the argument, which is a confirmation that we have an assumption.

Whatever might have caused Homo erectus to master fire doesn’t clarify who made those fires in Tanzania: Homo erectus or Homo ergaster?  Choice (B) is not correct.

Suppose Homo ergaster would have derived as much benefit from the master of fire as did the Homo erectus, or even more benefit.  That fact, by itself, would imply nothing about which one of these species created those fires in Tanzania. Denying this doesn’t change the validity of the argument.  Choice (C) is not correct.

Choice (D) is intriguing, because it may be true.  Both Homo erectus and Homo ergaster evolved from Homo habilis, so it’s quite likely that the Homo habilis was the sole source of cultural knowledge for either of these species.  BUT, we know that Homo erectus, presumably without the benefit of cultural knowledge about fire, was able to master fire.  If Homo erectus did that, why couldn’t Homo ergaster? In other words, the limits of the cultural knowledge inherited does not necessarily set limits on what these human species could achieve.   Therefore, we can draw no conclusion with respect to this argument.   Choice (D) is not correct.

If Choice (E) were true, it would support the argument, but a supporting statement is not necessarily an assumption.  We have to use the Negation Test.  Suppose Homo ergaster was all over in Tanzania, before & during & after the time that those fires were created.   Would that prove Homo ergaster started those fires?  Not necessarily.  It could still be true that both Homo ergaster and Homo erectus occupied that region, that only the latter had mastered fire, and therefore, that the later had to start those fires in Tanzania.  Thus, we can deny choice and it doesn’t necessarily contradict the argument.   Therefore, it is not an assumption.  Choice (E) is not correct.

 

10) The credited answer is choice (D).  If the town of Dryadia really does flood, then that’s the reason insurers won’t write flood insurance for it!  Therefore, the “only reason” cannot be “its similarity to Bayside in terms of where it is situated in the river valley.”  Choice (D), if true, obliterates the argument, so this is the best answer.

The argument say that “most insurers” don’t write flood insurance in either town, but if most don’t, this implies that some do.  Therefore, choice (A) is actually expected from the argument and does not challenge it at all.  Choice (A) is incorrect.

Choice (B) would not be surprising, and could be perfectly consistent with the argument.  We know Bayside had “catastrophic flooding“, but we don’t know for a fact that every single house was flooded—maybe or maybe not.  If some houses were not flooded, it sounds as if the insurers don’t write flood insurance for any house in Bayside, so even those houses that never flooded could not buy flood insurance.  Therefore, this would validate (B) without threatening the argument in any way.  Choice (B) is incorrect.

Choice (C) is irrelevant.  Even if no resident in absolutely any other town up and down the Katonic River Valley can buy flood insurance, that doesn’t necessarily shed light on why folks in a town in a completely different river valley can’t buy insurance.  Choice (C) is incorrect.

Choice (E) is too general and vague.  Yes, perhaps there are many ways a house can be flooded, and correspondingly, perhaps there are many reasons why an insurer would deny any particular house flood insurance.  Even if this is true, it doesn’t shed any light on exactly why the folks in Dryadia have trouble getting flood insurance.  Choice (E) is suggestive, but it doesn’t actually tell us anything.  Choice (E) is incorrect.

 

Editor’s Note: This post was originally published in April, 2012, and has been updated for freshness, accuracy, and comprehensiveness.

 

The post Introduction to GMAT Critical Reasoning appeared first on Magoosh GMAT Blog.
This Blog post was imported into the forum automatically. We hope you found it helpful. Please use the Kudos button if you did, or please PM/DM me if you found it disruptive and I will take care of it. -BB
Magoosh GMAT Instructor
Joined: 28 Dec 2011
Posts: 4450
Own Kudos [?]: 28569 [0]
Given Kudos: 130
GMAT Tuesday: OG Reading Comprehension #2 [#permalink]
Expert Reply
FROM Magoosh Blog: GMAT Tuesday: OG Reading Comprehension #2
Hello! We are back with the third in our series of 5 videos, diving into the reading comprehension section of the Official Guide to the GMAT. We are tackling another detail question, which means you’ll learn how to approach detail questions and the common wrong answers in detail questions.



Video Transcript
Hello. Welcome to the virtual reality edition of GMAT Tuesdays. I’ve got my virtual reality glasses. I’ve got my virtual reality headphones. And we’re ready to go. I couldn’t afford my own virtual reality goggles, so I made my own. Hi, we are diving into the official guide to the GMAT. We are moving on to question number two. In previous videos we looked at question number one and looked at the passage.

Passage Review
For a recap of the passage, cuz this is gonna be useful for this video. Remember that the main idea of the passage is we’re talking about two theories related to fish schooling. The structure of the first paragraph is the intro, second paragraph is theory A, third paragraph is critics of theory A plus the beginnings of theory B, the fourth paragraph is all about theory B or idea B. And then, the tone balanced-ish and the author is basically just presenting these two theories, okay.

Question #2
So if we take a look at question number two, we’re going to see that it is a detail question. Let me read the question for you real quick. The question says according to the passage, which is the key to knowing that it’s a detail question. Both theory A and theory B have been developed to explain how, and then we’ll finish that idea with the right answer.

So since this is a detailed question, we have specific steps that we want to follow. One, attack the passage which we did already. Two, we’re gonna rephrase the question, which we can do right now. Basically we’re looking at theory A and theory B and asking, what do they both explain?

What is the same between theory A and theory B? We’ll do our research on the passage to go find out, I know that in the beginning is for the introduction of these two theories. And what they are about, that’ll be a good place to look. I’ll anticipate an answer if you go the first paragraph, you’ll see they’re talking about schooling as a way to avoid predators.

So they answer choice, the correct answer should be something about. Avoiding predators and then finally, step five, we will eliminate any wrong answers until we arrive at the right answer. Also good to note is the common wrong answers with detailed questions, inferences. So any time that there’s an inference where you have to combine or synthesize ideas in the passage in a detail question that’s gonna be a wrong answer. Cuz we’re looking for things that are just stated specifically in the passage.

Next, a distortion, where they are taking information in the passage and tweaking it a little bit or moving it around, so that it’s not completely accurate. It might be half right and half wrong.

And then the new info, answer choices. Those are answer choices where they just introduce a new idea that isn’t actually in the passage.

Answer Choices
Okay, let’s dive into the answer choices. So answer choice [A] talks about fish hiding again. We saw in the first question, there was an answer choice related to fish hiding behind each other.

This is a great example of how the GMAT will sort of build up wrong answers across questions. So if you thought in the first question that the answer was fish hiding behind each other, you might jump to this wrong answer as well. So they are almost baiting you to choose two wrong answers in a row. But this is new information, there’s no talk of fish hiding behind each other or hiding at all, really, when they school. So that’s not right.

Answer choice B talks about protection. How schooling is meant for protection. And if you look in lines 4-6 in the introductory paragraph, that’s exactly what it said. It talks about schooling and as a way to protect themselves from predators.

Answer choice C says that this differs from other behavior, it might, but you don’t know cuz it’s not talked about in the passage. So this is new information that is not supported by the passage.

Answer choice D, talks about fish making rapid decisions. This also might be true but again there’s no information in the passage to support it, so you would want to eliminate it.

And finally answer choice E, says that schooling allows fish to survive around predators. And you could make an inference based on some on some information of passage in the fourth paragraph that since some fish do live where predators are, and they still school they must able to survive around predators.

But this is an inference you sort of strolled away from what is actually being stated in the passage. And so you know that it doesn’t work for a detail question. So the correct answer is B. All right, I hope you like this first edition of our virtual reality GMAT Tuesdays.

If you have any questions, leave comments down below. And of course, if you need anymore help head over to magoosh.com where there’s tons of friendly people like me ready to help you dominate test prep. All right, be excellent to the universe.



The post GMAT Tuesday: OG Reading Comprehension #2 appeared first on Magoosh GMAT Blog.
This Blog post was imported into the forum automatically. We hope you found it helpful. Please use the Kudos button if you did, or please PM/DM me if you found it disruptive and I will take care of it. -BB
Magoosh GMAT Instructor
Joined: 28 Dec 2011
Posts: 4450
Own Kudos [?]: 28569 [1]
Given Kudos: 130
Understanding Your GMAT Score Report [#permalink]
1
Bookmarks
Expert Reply
FROM Magoosh Blog: Understanding Your GMAT Score Report
When you take the GMAT, often at the test center itself you will get some of your GMAT score report as soon as you are done with your test.  You can choose to receive the GMAT total score report electronically or via snail mail (here’s a sample report from GMAC).  The electronic version will arrive by email within 20 calendar days.  The hard-copy report will be snail mailed within 20 calendar days, but given the vagaries of snail mail, may or may not arrive in your snail mail box within 20 calendar days.



You can take the GMAT more than once.  The total GMAT score report sent to adcom will include all GMATs you have taken in the past five years, except the ones you have canceled.  There is no trace of any cancellations on your score report (this last fact is new, a change from pre-2016 policy).

What is in the GMAT Score Report?
The total GMAT Score Report has the following components

1) Your Quantitative Score (0 – 60), with percentile

2) Your Verbal Score (0 – 60), with percentile

3) Your Total GMAT Score (200 – 800), with percentile

4) AWA Score (half-integers from 0 to 6), with percentile

5) Integrated Reasoning score (integer from 1 to 8)

Item #3, the “Total” score combines your Quantitative and Verbal scores, but doesn’t take any other parts into account.

Once again, as soon as you finish your GMAT in the test center, you will get almost the entire GMAT score report right away—everything except your AWA score, because that requires a human grader to review it.  The total GMAT score report arrives about 20 days later, finally including your AWA score.

What is a percentile?
The percentile associated with a particular score is the percent of the population whom you have outscored by getting that score.  For example, a total GMAT score of 700 is about the 89th percentile.  This means: if you score a 700 on your GMAT, you have done better than 89% of the folks who took the GMAT.  (The scoring has been relatively consistent for years, so GMAC can say: it’s not just 89% of the folks who took the GMAT when you took it, but 89% of everyone who took the GMAT in the past three years.)  Another way of saying that: scoring above 700 puts you in the top 10-11% of folks taking the GMAT.

A Verbal subscore of 40 would be in the 91st percentile, definitely in the top 10%.  By contrast, a Quant subscore of 40 would be only the 43rd percentile, not even in the top 50%!  The two subscores rare definitely not equivalent.  This in part reflects a vast asymmetry in the GMAT test-taking pool: many more GMAT takers in an international market excel in math and struggle in verbal, so commanding performances in math are reasonably common, whereas commanding performances in verbal are less frequent.

The charts below show the most recent GMAT percentile data provided by GMAC. The data comes from a sample of over 750,000 students who took the GMAT between 2013 and 2015.

GMAT Score Percentiles

PercentileScorePercentileScore

99%760-80034%520

98%75032%510

97%74030%500

96%73027%490

94%72025%480

91%71023%470

89%70020%460

86%69019%450

84%68017%440

82%67015%430

79%66014%420

76%65013%410

71%64011%400

69%63010%390

66%6209%380

62%6108%370

59%6007%360

56%5906%340-350

52%5805%330

49%5704%310-320

46%5603%280-300

43%5502%250-270

40%5401%220-240

36%5300%200-210

GMAT Scaled Score Percentiles

Quant PercentileQuant Scaled ScoreVerbal PercentileVerbal Scaled Score

97%5199%51

87%5099%50

77%4999%49

71%4899%48

65%4799%47

62%4699%46

59%4599%45

54%4498%44

52%4396%43

47%4296%42

45%4194%41

43%4091%40

39%3989%39

38%3885%38

36%3783%37

33%3681%36

29%3576%35

28%3472%34

26%3369%33

24%3267%32

21%3162%31

20%3060%30

18%2957%29

17%2852%28

14%2747%27

13%2644%26

12%2540%25

11%2437%24

10%2333%23

8%2231%22

8%2127%21

7%2023%20

6%1920%19

6%1818%18

4%1715%17

4%1613%16

3%1510%15

3%149%14

3%137%13

2%125%12

2%114%11

2%103%10

1%92%9

1%81%8

1%71%7

0%60%6

AWA and Integrated Reasoning Percentiles

AWA PercentileAWA ScoreIR PercentileIR Score

90%692%8

81%5.582%7

57%569%6

43%4.554%5

20%440%4

13%3.527%3

6%313%2

5%2.50%1

3%1.0-2.0

0%0-0.5

What is a “good” GMAT total score?
This is an impossible question to answer in general.  In some sense, the answer is: a “good” GMAT score is a score sufficient to help you get into the Business School that is right for you.  What makes a Business school “right” for you?  A panoply of factors, including location, cost, requirements, the feel of the school, etc. If you’re interested in knowing the average GMAT scores of top business programs, take a look at this helpful GMAT scores infographic.

Obviously, the higher the score, the more options you will probably have.  It may be that, to some extent, you can offset a lower college GPA with a stellar GMAT score report.

It is a fact that a solid test prep source, such as Magoosh, can raise your GMAT grade substantially.  In fact, Magoosh has a 50-point score increase guarantee: if you have already taken an official GMAT once, then Magoosh guarantees that if you use the product extensively, your score will increase by at least a minimum of 50 points (many users see much larger increases).  That’s extraordinary: such an increase can bring you from 650 (77th percent = top 23%) to 700 (89th percentile = top 11%)!

By all means, strive to do the best you can do, and use the effective help of Magoosh or another source of similar quality.  At the same time, it’s important to be realistic about your abilities and the time & energy you have to prepare.  If your first GMAT score was a 460, then with concerned effort and the support of Magoosh, you will be able to get up into the 500s and maybe even the 600s, but it may be that a GMAT score in the high 700s is unrealistic for you, and that’s OK.  Always strive for your personal best, but it’s hard to compete with everyone out there.   The goal of the GMAT is to get you into Business School, the goal of business school will be to get an MBA, and the goal of an MBA is to get into management positions in the business world.  Many folks who are wildly successful in upper management in the business world had less than stellar GMATs and went to unrecognizable un-prestigious business schools.  Conversely, some folks are brilliant test takers, and ace the GMAT, but then wind up not so successful in the rough and tumble of the business world.

A big part of success is being canny enough to know how to leverage your particular gifts to the greatest effect.  Trust the unique combination of gifts and talents you bring, seek to learn the skills that will most complement and bring forth who you are, and learn to recognize the environments in which you can most effectively thrive.  Do the best you can do on the GMAT, and trust that this will be good enough to lead you to where you need to be in the big picture.

 

Editor’s Note: This post was originally published in July, 2012 and has been updated for freshness, accuracy, and comprehensiveness.

 

The post Understanding Your GMAT Score Report appeared first on Magoosh GMAT Blog.
This Blog post was imported into the forum automatically. We hope you found it helpful. Please use the Kudos button if you did, or please PM/DM me if you found it disruptive and I will take care of it. -BB
Magoosh GMAT Instructor
Joined: 28 Dec 2011
Posts: 4450
Own Kudos [?]: 28569 [0]
Given Kudos: 130
Common GMAT Topic: Descriptive Statistics [#permalink]
Expert Reply
FROM Magoosh Blog: Common GMAT Topic: Descriptive Statistics
Here’s a whirlwind tour of one of the most frequently tested sets of concepts on the GMAT Quantitative section: GMAT statistics. You don’t have to be an expert on statistics: a little goes a long way. Let’s learn some basic descriptive statistics formulas and go from there.



Descriptive Statistics Formulas
The Mean
One very important GMAT Statistics idea is the mean.  The mean is just the ordinary average: add up all the items on the list, and divide by the number of items.  As a formula, that’s



Notice, we can rewrite that as: (sum of items) = average*(number of items).  Rewritten in sum form, it becomes one of the most powerful statistics formulas on the GMAT.  For more exotic kinds of averages you may see on descriptive statistics on the GMAT, see this post on Weighted Averages.

The Median
Another important GMAT Statistics idea is the median.  When we put the list in ascending order, the median is the middle.  If there are an odd number of items on the list, the middle item equals the median: for example, in the seven-element set {3, 5, 7, 9, 13, 15, 17}, the median is the fourth number, 9.  If there are an even number of items on the list, then the median is the average of the two middle numbers; for example, in the eight-element set {3, 5, 7, 9, 13, 15, 17, 17}, the median is 11 (the average of the fourth & fifth entries, 9 and 13).  Notice: when the number of items on the list is even, the median can equal a number not on the list.  Numbers above and below the median can be equal to the median, and that doesn’t change the fact that it’s a median; for example, the median of the set {1, 3, 3, 3, 3, 3, 74, 89, 312} is just 3, the fifth number of that nine-element set.

Also, notice that an absurdly large number, far away from the rest of the set, such as 312 in this last set, has zero effect on the median, although it would have a big effect on the mean.  As long as the middle numbers stays in the middle, changes to the values of the outer numbers has no effect on the median; by contrast, changing any number in the set changes the mean.

Both the mean and the median are “statistical measures of center”—that is, they are different ways of responding to the question “which number is most representative of the set?”  The median simply “chunks” the set, whereas the mean “feels” each number.  Changing  the value a single number, especially toward the high or low end of the set, never changes the median and always would change the mean.

Range
The GMAT loves this statistical measure, because it’s so simple.  The range is the difference between the maximum value and the minimum value.  In the set {3, 5, 7, 9, 13, 15, 17}, the range = 17 – 3 = 14.  In the set {1, 3, 3, 3, 3, 3, 74, 89, 312}, the range = 312 – 1 = 311.  Notice that the range, unlike the mean or median, is always positive (it would be zero if the max and min were the same number!)  For the range, the only thing that matters is the top and bottom values in the set: those two values are the only two numbers that the range “feels.”  We could change the values of almost every number in the middle of the set, and the range would not change at all.

Standard Deviation
The range is a statistical measure of the spread from the highest to the lowest value, but it doesn’t “feel” the numbers in between.  The standard deviation is also a statistical measure of spread, that is to say, an indication of how far apart the numbers on the list are from each other.  Like the mean and unlike the range or median, the standard deviation “feels” every number on the list.   It has a technical definition that we can forego here; the majority of appearances of standard deviation on the GMAT revolve around a few simple ideas about it.

a) Like the range, the standard deviation is always zero or positive, never negative.

b) If all the entries of the list are equal, the standard deviation = 0.  In other words, they don’t deviate at all, because they’re all the same. For GMAT statistics, you should know that this is the only way to get a minimum standard deviation of zero.

c) If you add/subtract a constant to/from every number on a list, that doesn’t change the standard deviation at all.  It’s just like taking the batch of data points and sliding them up or down the number line: that process doesn’t change how far apart they are from each other.

d) If you multiply/divide a list by a constant, then you also multiply/divide the standard deviation by this constant.

I’ll add an additional rule that really could only come into play in a very difficult upper-700s question of GMAT statistics:

e) If all the entries arethe same distance from the mean, that distance is the standard deviation.  For example, in the set {3, 3, 3, 7, 7, 7}, the mean = 5, and every number “deviates” from the mean by exactly two units, so the standard deviation = 2.

See this post for more on standard deviation.

GMAT Statistics Practice
If you master these simple ideas, you will dominate on Descriptive Statistics in the GMAT Quantitative section.  Here are some GMAT statistics practice questions:

1) https://gmat.magoosh.com/questions/112

2) https://gmat.magoosh.com/questions/938

3) https://gmat.magoosh.com/questions/937

 

Editor’s Note: This post was originally published in May, 2012 and has been updated for freshness, accuracy, and comprehensiveness.

 

The post Common GMAT Topic: Descriptive Statistics appeared first on Magoosh GMAT Blog.
This Blog post was imported into the forum automatically. We hope you found it helpful. Please use the Kudos button if you did, or please PM/DM me if you found it disruptive and I will take care of it. -BB
Magoosh GMAT Instructor
Joined: 28 Dec 2011
Posts: 4450
Own Kudos [?]: 28569 [0]
Given Kudos: 130
GMAC’s Official IR Practice Questions [#permalink]
Expert Reply
FROM Magoosh Blog: GMAC’s Official IR Practice Questions
Back in June 2012, the GMAT launched the Integrated Reasoning (IR) question.  At the time, there was a panic among GMAT test-takers, and GMAC saw one of the largest peaks ever in number of GMATs taken, just before June, 2012—all the folks who wanted to avoid this then-brand-new question!  In 2013 and 2014, adcom saw a large number of these pre-IR GMATs, but by 2016, the number of these has dropped significantly.  Almost every official GMAT adcom sees now is one with Integrated Reasoning.  GMAT integrated reasoning sample questions and IR practice are now an important piece of GMAT prep. This question is here to stay.



GMAT IR Practice: Question Formats
The GMAT Integrated Reasoning question formats are highly dependent on technology, and unlike traditional Verbal and Quantitative questions, these would be too compromised in a print format. Therefore, from the beginning, GMAC never put official Integrated Reasoning questions in the GMAT Official Guide: the official questions always have been online.   At first, a code in the back of the GMAT Official Guide would give you access to a separate site that just had the official Integrated Reasoning questions.

Starting with the OG 2016, GMAC put all the questions in the OG online, in case you wanted to study them in an online format rather than in print format.  When GMAC made this move, logically they put the 50 official Integrated Reasoning questions on the same website with the other questions.   The code in your copy of the OG will give you access to that website.   The Quant & Verbal questions on that website are identical to those in the printed OG, but going to that website is the only way you can practice official Integrated Reasoning questions.

GMAT Integrated Reasoning Sample Questions, Types and Terms
Each IR question is one of four formats:

(1) Multi-Source Reasoning (MSR)

(2) Table Analysis (TA)

(3) Graphics Interpretation (GI)

(4) Two-Part Analysis (2PA)

If these are unfamiliar to you, you can read about the basics in the Magoosh IR eBook.  The official website and the actual GMAT itself appear to give questions of the four formats successively: that is, all the MSR, then all the TA, then all the GI, then all the 2PA.  This is in sharp distinction to the Quant & Verbal sections, in which quant formats (PS & DS) or verbal formats (RC & CR & SC) are freely interspersed.

We do know: eight of the 12 questions will count, and the other four will be experimental questions.  We do know: whatever block of 8 questions counts, those 8 questions will have the same distributions of question for all test takers.  We don’t know what that distribution is: let’s say, just for simplicity, that it’s 2 MSR questions, 2 TA questions, 2 GI questions, and 2 2PA questions.  That mix will count for everyone.  Now, the four experimental questions don’t count, so they could be any mix of problems: it could be one of each type, or two of one type and two of another, or all four of a single type.  Suppose Fred happens to get an IR section on which all four of the experimental questions are 2PA.  In the course of the 12 questions, Fred would see six 2PA questions: if he were counting, he would realize that the experimental questions would have to be 2PA, but think about it.  He doesn’t start to realize something is unusually until he gets to his 4th or 5th or even 6th 2PA.  At that point, he knows some of those 2PA questions had to be experimental, but he has no way of knowing which two count: the first two? the last two? the third and the fifth?  Of course, of six questions, there are 6C2 = 15 different ways the two that count could be distributed among them, and all 15 of those scenarios are equally likely.  The upshot is: even if you have an inkling that you are getting more of this question than that, you always have to treat the question in front of you as if it counts.

Also, let’s talk a little about this word “question”—we say the website has 50 practice “questions” and the IR section has 12 “questions,” but I would argue a more accurate term would be “screens.”  The website has 50 screens, and the IR section of the GMAT has 12 screens.  Each screen will be one of four formats—MSR, TA, GI, 2PA—and almost every screen has more than one question on it.  Rather than talk about how many questions within each question, for clarity, for the remainder of this post, I will refer to each “screen” and how many questions on that screen.  Here’s a little of what we can glean about the four screen formats from the 50 IR practice screens on the website.

Finally, remember that there is no partial credit on the GMAT Integrated Reasoning.  If there are multiple tasks on a single IR screen, you must get every single task on the screen correct to earn credit for that screen.

Multi-Source Reasoning
This is the only of the four types in which the same content appears across multiple screens.  For every other question format, all the relevant content appears on one screen, and none of that content appears on any other screen.

The information for a bank of MSR questions appears in separate “cards” on the left side of a split screen, and the questions appear on the right side, much like Reading Comprehension.   The same set of cards is present for 3-5 MSR screens, again, much like a RC passage.   At least one card is all text, and sometimes all the cards are all text.  A card may also contain a graph, a chart, a table, or even a mathematical formula relating ideas discussed on other cards.  As a general rule, most of the questions can only be answered by combining information from different cards.

Some MSR screens have a single five-choice multiple choice (MC) question: on the whole Integrated Reasoning section, this is the only screen type, the only question type, on which there is only one question on the screen.

The others, in fact, the majority of MSR screens, have what I call Multiple-Dichotomous Choice (MDC) question designs.  A dichotomous choice question is one that has only two possible responses (yes/no, true/false, etc.)  The GMAT Integrated Reasoning MDC design has three such questions in a table.  Here’s an example of a frivolous MDC, just so you can see the design.



Each row is a dichotomous choice: we have to give one of only two choices as a response.  Of course, the answers to this playful MDC are “helps” in the first two rows, and “doesn’t help” in the last row.  This MDC question depended on general knowledge, whereas the MDC questions that appear in the MSR format will depend on the information on the cards.  If the IR screen contains a set of MDC questions, you must get all three correct to get credit for that screen.

Here’s an Integrated Reasoning Multi-Source Reasoning practice question.

Table Analysis
Each IR Table Analysis screen contains a verbal prompt and a sortable table, that is, a table you can put into ranked order by any column.   Tables typically have 5-8 columns.   The GMAT loves to have some columns that give data about ranking: for example, the table could have one column that lists the actual areas, in sq miles, of counties, and another that lists the rank of the area in terms of the world—the United States of America has an area of 3.8M sq miles and rank of 4th in the world.

Every table appears just once, on one screen, with a set of MDC questions, always three individual questions, as above.   Once you answer those three questions and hit submit, that particular table is gone forever.

Here’s an Integrated Reasoning Table Analysis practice question.

Graphics Interpretation
Each GI screen presents a verbal prompt and a graph.  The questions on a GI screen are in the form of two drop-down menus in a fill-in-the-blank format.  That is to say, underneath the graph will be one or two sentences, with a total of two blanks: the student “fills in” the blank with a choice from the drop-down menu: the drop-down menus have 3-5 choices.

The graphs can be of several different types, including bar charts (including clustered column and segmented column charts), scatterplots, and bubble charts.  One more exotic type of chart is the numerical flowchart.   Here’s a post with a single Integrated Reasoning Graphics Interpretation practice question, and here’s a set of IR GI practice questions.

Two-Part Analysis
This question format presents some kind of prompt, and then a table.  The table has three columns.  At the top of the first two columns are the “questions,” and the possible answers to the questions are listed in the third column.  The same set of possible answer choices applies to both questions.  Often, the two questions are connected, in the sense that one must answer one to figure out the answer to the other.

In some ways, 2PA is the most flexible of the four question formats.  Individual 2PA screens can be purely verbal (similar to RC and CR questions) or mathematical.

This post gives a single Integrated Reasoning Two-Part Analysis practice question, and here’s a set of IR 2PA practice questions.

Official Integrated Reasoning GMAT Practice: Summary
The GMAT Integrated Reasoning questions are hard.  Magoosh has a GMAT IR ebook as well as a full bank of lessons and practice questions to prepare you.  Together, we can do this! 

See also: Is the GMAT Integrated Reasoning More Important Now?

 

Editor’s Note: This post was originally published in July, 2012 and has been updated for freshness, accuracy, and comprehensiveness.

 

The post GMAC’s Official IR Practice Questions appeared first on Magoosh GMAT Blog.
This Blog post was imported into the forum automatically. We hope you found it helpful. Please use the Kudos button if you did, or please PM/DM me if you found it disruptive and I will take care of it. -BB
Magoosh GMAT Instructor
Joined: 28 Dec 2011
Posts: 4450
Own Kudos [?]: 28569 [0]
Given Kudos: 130
Number Sense for the GMAT [#permalink]
Expert Reply
FROM Magoosh Blog: Number Sense for the GMAT
What is number sense and how can you recognize number sense problems on the GMAT? Before we get into the details, let’s start with a few number sense practice problems. Remember, no calculator.



Warm-Up Problems


1) Rank those three in order from smallest to biggest.

(A) I, II, III

(B) I, III, II

(C) II, I, III

(D) II, III, I

(E) III, I, II

 

2) Let P = 36000.  Let Q equal the sum of all the factors of 36000, not including 36000 itself.  Let R be the sum of all the prime numbers less than 36000.  Rank the numbers P, Q, and R in numerical order from smallest to biggest.

(A) P, Q, R

(B) P, R, Q

(C) Q, P, R

(D) R, P, Q

(E) R, Q, P

 



3) Rank those three in order from smallest to biggest.

(A) I, II, III

(B) I, III, II

(C) II, I, III

(D) II, III, I

(E) III, II, I

Solutions for these number sense problems will come at the end of this blog article.

What is Number Sense?
Many GMAT Quantitative Problems, like the foregoing pair, test number sense.  What is number sense?  Number sense is a good intuition for what happens to different kinds of numbers (positive, negative, fractions, etc.) when you perform various arithmetic operations on them.

Number sense is what allows some folks to “see” shortcuts such as estimation or visual solutions.  For example, in any of the problems above, there’s absolutely no need to do any detailed calculations: in fact, folks with number sense can probably do all the math they need to do in their heads.

Examples of a few number sense facts
  • Making the numerator of a fraction bigger makes the whole fraction bigger.
  • Making the denominator of a fraction bigger makes the whole fraction smaller.
  • (big positive) + (small negative) = something positive
  • (small positive) + (big negative) = something negative
  • Multiplying by a positive decimal less than one makes something smaller.
  • Dividing by a positive decimal less than one makes something bigger.
Of course, it would be near impossible to make anything like a complete list.  The left-brain reductionist dreams of something like an exhaustive list one could study, but number sense is all about right-brain pattern matching.   If you’re not familiar with the distinction of left/right hemisphere, see this GMAT post which touches on similar issues.

How do you get number sense?
If you don’t have it, how do you get it?  That’s not an easy question.  There’s no magical shortcut to number sense, but here are some concrete suggestions.

1. Do only mental math.  You shouldn’t be using a calculator to practice for the GMAT anyway.  Try to do simpler math problems without even writing anything down.   Furthermore, look for opportunities every day, in every situation, to do some simple math or simple estimation (e.g. there are about 20 cartons of milk on the grocery store’s shelf—about how much would it cost to buy all twenty?)

2. Look for patterns with numbers.  Add & subtract & multiply & divide all kinds of numbers—positive integers, negative integers, positive fractions, negative fractions, and look for patterns. Number sense is all about pattern with numbers!!

3. This is a BIG one—in any GMAT practice problem that seemed (to you) to demand incredibly long calculations, but which had a very elegant solution of which you would have never dreamt—that problem & its solution are pure gold.  In a journal, write down what insights were used to simplify the problem dramatically.  Force yourself to articulate this, and return to this solution and to your notes on it often.  Over time, you should develop an array of problems like this, and if you study those solutions, you probably will start to see patterns.

4. Similar to #3: search the two forums,GMAT Club and Beat the GMAT, for similarly difficult questions, and look for elegant solutions.  That’s a great place to ask the experts (including yours truly) for more detailed explanations of their choices in the solution.

5. Here’s a variant on a game you can play, alone or with others who also want practice.  Pick four single digit numbers at random—some repeats are allowed.   You could roll a die four times, and use the results.  Now, once you have those four numbers, your job is to use all four of them, each of them only once, and any arithmetic, to generate each number from 1 to 20.  By “any arithmetic,” I mean any combination of: (a) add, subtract, multiply, divide; (b) exponents; and (c) parentheses & fractions

For example, if the four numbers I picked were {1, 2, 3, 4}, I could get 2 from



For any one number, you only need to come up with it in one way (although you can consider it a bonus to come up with multiple ways for a single number!)  Here, I show three ways just to demonstrate the possibilities.  A few examples for some of the higher numbers:



Notice that I used a variant of the expression for 13 to create an expression for 14.  Also, if I changed the plus sign in the expression for 13 to a minus sign, I would get an expression for 11.  Also, notice that if 1 is one of the four numbers, then if you don’t need it, you can simply multiply by it; furthermore, notice in the expression for 13 and in the second expression for 2 above, the exponent of 1 is a useful place to stash other numbers you don’t need!

As you practice, you will start to develop a sense of how expressions for one number can be tweaked to give you another number.   Overall, using similar combinations, you have to get every number from 1-20 with these four, or with whatever four you pick.   Actually, the set {1, 2, 3, 4} is a very good warm-up set.  When you want more of a challenge, use {2,3,3,5}.   

One of our Remote Test Prep Experts, Jeff Derrenberger, created an awesome web game based on this mental math game. Click on the banner to check it out!



Practice Number Sense Problem
Here’s a practice number sense problem. If you didn’t get anywhere with the practice problem, you may want to study the solution below carefully.

4) https://gmat.magoosh.com/questions/54

Practice Problem Solution
1) Notice that all three of these are close to fractions that equal 1/3.  The fractions that equal 1/3 would be, respectively, 50/150, 110/330, and 300/900.   First of all, only the second one has a higher numerator, so the second one is more than 1/3 and the other two are less than 1/3.  Therefore, II is the greatest.

Now, from I and III, which is greater?  Well, think about it this way.  50/150 = 300/900, because both of those equal 1/3.   How much less than one third is each one of these?  Well, 47/150 is 3/150 less than 50/150 = 1/3, and 299/900 is 1/900 less than 300/900 = 1/3.  Well, clearly, 3/150 > 1/900 (the latter has a smaller numerator and a larger denominator!)  Therefore, starting from 1/3, 47/150 goes down further than does 299/900.  Therefore, 47/150, dropping down a larger distance, must be the minimum value.  Therefore, the correct order is I, III, II.

Answer = (B)

 

2) We know that some of the factors of 36,000 are 18,000, 12,000, and 9,000.  Right there, those three add up to 39,000 more than 36,000.  Right there, we know that P < Q.  We can eliminate (C) and (E).

Now, R is a little trickier.  We don’t need to have detailed knowledge here.  We know there are several prime numbers less than 100.  Obviously, the density of prime numbers gets slightly less as we get bigger.  Let’s assume, extremely conservatively, that when we get up into the 20 and 30 thousands, there is at least one prime number every thousand: one between 20K and 21K, one between 21K and 22K, all the way up to 36K.  The 6 primes in the thirty thousands are all greater than 30K, so let’s estimate their sum as (30K)*6 = 180K.  The 10 primes in the twenty thousands are all greater than 20K, so let’s estimate their sum as (20K)*10 = 200K.  Right away, that’s 380K on an extremely conservative estimate.  There is no way that the sum of the factors of 36K, not including 36K itself, is going to be more than ten times 36K!  Thus, R is much larger than Q, and the correct order is P < Q < R.

Answer = (A)

 

BTW, if your curious, according to Wolfram Alpha, the sum of the factors of 36000, not including 36000, is Q = 91,764, and the sum of all the prime number less than 36000 is R = 64,711,067.

 

3) Here, we have to “un-simplify” the square-roots to get a sense of their relative size.



From this, we see that II is less than I.  From this alone, we can eliminate (A) & (B).

The trickier item on the list is III.  Without a calculator, it would be nearly impossible compute an exact value for the fourth-root of 401.  But consider this: the fourth root of a number is the number to the power of 1/4, and 1/4 = (1/2)*(1/2), so the fourth root is the square root of a square root.  Now, of course, 401 is not itself a perfect square, but it is very close to a perfect square.



This demonstrates that III. is slightly larger than I.   Therefore, the order from least to greatest is II, I, III.

Answer = (C)

 

Editor’s Note: This post was originally published in December, 2012, and has been updated for freshness, accuracy, and comprehensiveness.

 

The post Number Sense for the GMAT appeared first on Magoosh GMAT Blog.
This Blog post was imported into the forum automatically. We hope you found it helpful. Please use the Kudos button if you did, or please PM/DM me if you found it disruptive and I will take care of it. -BB
Magoosh GMAT Instructor
Joined: 28 Dec 2011
Posts: 4450
Own Kudos [?]: 28569 [0]
Given Kudos: 130
How to Get a Perfect GMAT Score [#permalink]
Expert Reply
FROM Magoosh Blog: How to Get a Perfect GMAT Score
The sub-4 minute mile; a perfect round of golf; a 300 in bowling…oh the elusive Mt. Everest. A GMAT 800: the perfect GMAT score.

Improbables of perfection, how can any ever hope to summit your slippery slopes?

Hard work won’t quite cut it. An element of luck, a touch of madness, and more than an average serving of raw talent might be enough.

If you’re aiming to get a GMAT perfect score, check out this SlideShare below, and read the following tips.



Perfect GMAT Score
In terms of attainability, where then does the GMAT perfect score fall – that perfect 800 we wish we could have emblazoned on our transcript? Well, fewer than 30 each year attain this coveted score. And as I am sure a few of these are tutors, that means even fewer test takers are scoring a perfect 800 on the GMAT.

To put things into perspective, 150 people climb Everest each year (though 5 do die – which is something I’ve never heard befall a GMAT test taker).

Clearly, a perfect score is extremely difficult – though not necessarily impossible. Below are some points to keep in mind if you have your eye on GMAT perfection.

GMAT Perfect Score: Some Criteria to Consider
Educational Background
The philosopher-physicist has it best. Strong quant skills coupled with formidable problem skills, this person is able to quickly see the big picture and discern the answer, that one tree in the dense forest of verbiage and equivocation.

Physicists are also adept at seeing some of the underlying patters in the verbal section. But it is the philosopher who truly excels. The critical reasoning section? Philosophers have been doing this kind of analytical thinking for years – and the level of critical reasoning philosophers have been engaging in makes the GMAT CR seem like a high school equivalency exam.

While the philosopher-physicist hybrid is the ideal, there are other fields of study that also handle the GMAT well. If you have a computer science background, you will have the problem solving skills necessary to do very well on the test. In general math types will fare well; even the verbal section is based on logic (think Sentence Correction). Yet, math types out there might want to make sure they are avid readers. If they have a penchant for literature, growing up with the likes of Dickens and Steinbeck, they will likely do well on the verbal. Ultimately, the student who will do best is one who blends analytical and mathematical thinking with a healthy dollop of verbal panache — whether they be an English major who does Sudoku in their spare time or an engineer with a thing for Jane Austen.

Avid Reading
Those who read challenging books and articles should have little problem navigating the dense, quasi-academic language of the GMAT. Proper idiomatic construction should flow from their tongues with the same insouciant ease most of us have discussing the weather. Grammatical niceties will jump out from the page, amidst the noise of GMAT distractors. (I say this based on my literary-inclined friends who find that they can rely on their ear to ace the Sentence Correction section — something even most native English speakers can’t do).

My advice: read challenging – but not utterly boring – works. And pay attention to language and turns of phrases. Additionally, it will help if you read widely. An article on species extinction in The New Yorker for breakfast and post-lunch reading of the latest Booker Prize-winning novel.

Meticulousness
The perfect scorer isn’t necessarily the fastest. But they are the most careful. The perfect scorer double-checks their answers and does not leap at an answer choice because it sounds good. He/she is aware of the traps that the GMAT lays, and is assiduously careful to avoid falling for them. Indeed, even perfect scorers aren’t always perfect scorers. Perhaps on their mock exams they missed a few questions.

Likely, once they saw their answer was incorrect, they could go back to the question and identify what was wrong with it. There mistake wasn’t conceptual; it was careless.

When you get close to perfection on the GMAT, it is likely meticulousness keeping you from an 800. Double-check that answer on Data Sufficiency (that notoriously and deceptively tricky section), don’t gloss over an answer choice on Reading Comprehension and end up missing that one critical word that invalidates the answer choice.

Meticulousness carries over to other facets of the GMAT besides answering questions. How you prepare is key to improving your score, especially when that score is the difference between 750 to 800 (those extra 50 points will require an extra serving of self-discipline.)

The perfect 800 scorer will be highly regimented and devise a GMAT study schedule. They will focus on their weaker areas and be as precise as possible when determining exactly why they missed those precious few points keeping them from scoring an 800. “I’m bad at hard science passages” is not the kind of mindset you need. Rather, “I will make sure to pay attention to how the author slips in a technical term mentioned earlier in the passage, instead of trying to justify an answer that I know is not quite right because I’m looking at the wrong part of the passage”. Essentially, perfect scorers are sticklers who look out for everything, from small math errors to grammatical nuances.

The Best Test Prep
To be able to attain a perfect score on the GMAT, you have to understand the subtleties of the test, and only official practice material will truly give you that. Yes, there is other material out there, some of which does a good job — Magoosh GMAT (yes, I know, conflict of interest), Manhattan GMAT (practice tests are even harder than the GMAT), and Veritas (does an okay job) — and there are the rest, from the benignly bad to the downright abominable, which will actually hurt your score.

This may be the first time you are hearing about Magoosh, and while we definitely can’t promise you a perfect score, we can promise you the best online video prep. You will nail the fundamentals (even if you weren’t a philosopher poet) and will have hundreds of practice questions to help you scale the Mt. Everest that is the GMAT.

How to Get a Perfect Score on the GMAT [/b] from Magoosh

 

Editor’s Note: This post was originally published in February of 2012 and has been updated for freshness, accuracy, and comprehensiveness.

 

The post How to Get a Perfect GMAT Score appeared first on Magoosh GMAT Blog.
This Blog post was imported into the forum automatically. We hope you found it helpful. Please use the Kudos button if you did, or please PM/DM me if you found it disruptive and I will take care of it. -BB
GMAT Club Bot
How to Get a Perfect GMAT Score [#permalink]
   1   2   3   4   5   6   7   8   9   10   11   12   13   

Powered by phpBB © phpBB Group | Emoji artwork provided by EmojiOne